Feb 22, 2011

final coaching

TRACHEOSTOMY
1. What is the best position when suctioning the client with tracheostomy?
a. High Fowler’s Position
b. Flat in Bed
c. Semi-Fowler’s Position
d. Trendelenberg Position
Rationale: C – Davis’s NCLEX RN – Success Second Edition 539
2. What is incorrect when suctioning the patient?
a. Apply suction when inserting suction catheter.
b. Do not apply suction when inserting suction catheter
c. Use strict aseptic technique
d. Use clean Gloves
Rationale: B – Davis’s NCLEX RN – Success Second Edition 539
3. Apply suction intermittently for how long?
a. 2 – 3 minutes
b. 4 – 8 minutes
c. 5 – 10 seconds
d. 1 – 3 seconds
Rationale: C – Davis’s NCLEX RN – Success Second Edition 539
4. Before suctioning the client with tracheostomy administer how many % of oxygen?
a. 90%
b. 50%
c. 70%
d. 100%
Rationale: D – Davis’s NCLEX RN – Success Second Edition 539
5. What is the main goal after the patient has under gone tracheostomy?
a. Health teaching
b. Maintain patent airway
c. Improve nutritional status
d. Alleviate apprehension
Rationale: B – Davis’s NCLEX RN – Success Second Edition 539
6. When cleaning the non-disposable inner cannula you use?
a. Clean gloves
b. Bare hands
c. Sterile Gloves
d. None of the above
Rationale: C – Nurse’s Quick Check Skills 483
7. What is the recommended mm Hg of the cuff pressure?
a. 18 mm Hg
b. 25 mm Hg
c. 12 mm Hg
d. 30 mm Hg
Rationale: A – Nurse’s Quick Check Skills 484
8. What is the incorrect use of a Cuffed Tracheostomy Tube?
a. Provide and maintains a patient airway
b. Prevents aspirating food or secretions
c. Removal of tracheobronchial secretions
d. Don’t use positive pressure ventilation
Rationale: D – Nurse’s Quick Check Skills 486
9. What should you prepare in the clients bed side? SATA
a. Suction equipment
b. Clean obturator
c. Sterile tracheostomy
d. Sterile tracheal dilator
e. Sterile haemostat
f. Sterile inner cannula
Rationale: ACDEF – Nurse’s Quick Check Skills 487
10. In re-inserting the tracheostomy tube what is to be avoided? SATA
a. Tracheal trauma
b. Perforation
c. Compression
d. Asphyxiation
e. All of the above
f. None of the above
Rationale: ABCD – Nurse’s Quick Check Skills 487

Communicable Disease
1. It is a spirochetal illness. Most common tick-borne infectious disease in United States. What kind of infectious disease is this?
a. Bird flu
b. Monkey pox
c. Lyme disease
d. Salmonella
Rationale: B – Davis’s NCLEX – RN Success Second Edition 544
2. It is highly infectious virus affecting millions of birds across Asia
a. Avian influenza
b. Rabies
c. Tularemia
d. West nile virus
Rationale: A – Davis’s NCLEX – RN Success Second Edition 547
3. How many days of onset have the Monkey Pox?
a. 24 – 72 hour
b. Within 24 hours
c. 7 – 18 days
d. 3 – 14 days
Rationale: C – Davis’s NCLEX – RN Success Second Edition 544

2. What are the symptoms if you have rabies?
a. Abdominal pain
b. Dysphagia
c. Eye infection
d. Severe diarrhea
Rationale: B – Davis’s NCLEX – RN Success Second Edition 544

3. What is caused by the infectious disease West Nile Virus?
a. Drinking contaminated water
b. Eating the meat from cattle
c. Poultry
d. Mosquitoes
Rationale: D – Davis’s NCLEX – RN Success Second Edition 544

4. What is the cause of Anthrax?
a. Clostridium botulinum
b. Yersinia Pestis
c. Bacillus Anthracis
d. Francisella Tularensis
Rationale: C – Davis’s NCLEX – RN Success Second Edition 548

5. What is the early stage of signs and symptoms of Hantavirus Pulmonary syndrome?
a. Cough
b. SOB
c. Chest tightness
d. Muscle aches
Rationale: D – Davis’s NCLEX – RN Success Second Edition 548

6. Where does it rarely bleed if a client has a Viral Hemorrhagic Fever?
a. Ears
b. Organs
c. Mouth
d. Under the skin
Rationale: A – Davis’s NCLEX – RN Success Second Edition 548

7. If you have a Septicemic Plague what are the signs and symptoms? SATA
a. Fever
b. Chills
c. Bleeding into skin
d. Bleeding into organs
e. Shock
f. None of this
Rationale: ABCDE – Davis’s NCLEX – RN Success Second Edition 548

8. What is our drug of choice if you have Rabbit Fever (Tuleremia)?
a. Gentamicin
b. Doxycycline
c. Ciprofloxacin
d. Streptomycin
Rationale: D – Davis’s NCLEX – RN Success Second Edition 548


Postoperative
1. What is the complication during Postoperative care? SATA
a. Atelectasis
b. Arrhythmias
c. Hypertension
d. Hypervolemia
e. Septicemia
f. Septic Shock
Rationale: ABEF – Nurse Quick Check Skills 382
2. How many hours will the client recover from the spinal anesthesia?
a. 4 hours
b. 6 hours
c. 8 hours
d. 24 hours
Rationale: B – Nurse Quick Check Skills 382
3. If patient is in a skeletal traction what do you not assess for?
a. LOC
b. Skin color
c. Mucous membrane
d. Eye color
Rationale: D – Nurse Quick Check Skills 382
4. What is not a complication during Postoperative care?
a. Wound infection
b. Thrombophlebitis
c. Pneumonia
d. Diarrhea
Rationale: D – Nurse Quick Check Skills 382
5. Patient varies in the different ways recovery time. How do they vary?
a. Amount of fats
b. Lifestyle
c. Dosage
d. Premedication regiment
Rationale: B – Nurse Quick Check Skills 382
6. Which is not true as a nursing diagnosis for the patient who is in a postoperative care?
a. Ineffective breathing pattern related to general anesthesia
b. Risk for infection related to disruption of skin integrity
c. Altered tissue perfusionrelated to shock
d. Diarrhea related to increase peristalsis
Rationale: D – Davis’s Nclex – RN Success Second Edition 553
7. Which should be immediately reported to the physician?
a. 45 beats/min
b. 110 beats/min
c. 60 beats/min
d. 70 beats/min
Rationale: A – Davis’s Nclex – RN Success Second Edition 553
8. What will be the incorrect nursing care plan for patient who had developed Atelectasis?
a. Position at the affected side
b. Cough and deep breathing exercises
c. Force fluids
d. Postural drainage
Rationale: A – Davis’s Nclex – RN Success Second Edition 554
9. What is the nursing action if patient has pneumothorax during the Postoperative Care? SATA
a. High Fowlers Position
b. Assist Thoracentesis
c. Chest Xray
d. Assist with chest insertion
e. Flat in bed
f. Vital Signs
Rationale: ABCDF – Davis’s Nclex – RN Success Second Edition 553
10. If ever the patient have a wound evisceration what would be the cause of it? EXCEPT:
a. Hiccups
b. Vomiting
c. Obesity
d. Narcotics
Rationale: D – Davis’s Nclex – RN Success Second Edition 556

PERIOPERATIVE
1. During the perioperative care you educate the client to do diaphragmatic breathing exercises. How many seconds does the client need to hold his or her breath?
a. 1 – 3 seconds
b. 3 – 5 seconds
c. 5 – 8 seconds
d. 8 – 11 seconds
Rationale: B – Davis’s NCLEX-RN Success Second Edition 550
2. Which is incorrect when doing the coughing exercise?
a. Leaning forward slightly from a sitting position.
b. Inhale and exhale slowly.
c. Mouth should be open widely
d. “huff” cough to stimulate the cough
Rationale: C – Davis’s NCLEX-RN Success Second Edition 550
3. During skin preparation what is a correct nursing action?
a. Check for allergy to iodine.
b. Hexachlorophene should be left on the skin for 15 mins.
c. Wash skin with soap before using Zephiran (Benzalkonium Cl)
d. Shaving with a razor is more preferred to Depilatory creams or clipping of hair.
Rationale: A – Davis’s NCLEX-RN Success Second Edition 550
4. What is the incorrect perioperative care for Gastrointestinal surgery?
a. NPO for 6 – 8 hours
b. High pressure setting with Levine tube
c. Low pressure setting with Intestinal tube
d. High pressure setting with Salem-sump
Rationale: B – Davis’s NCLEX-RN Success Second Edition 551
5. How many minutes do you administer the preoperative drugs before the anaesthetic induction?
a. 30 mins
b. 35 mins
c. 65 mins
d. 85 mins
Rationale: C – Davis’s NCLEX-RN Success Second Edition 551
6. What is not a fatal reaction of a client during latex allergy?
a. Hypertension
b. Flushing
c. Diaphoresis
d. Cramping
Rationale: A – Davis’s NCLEX-RN Success Second Edition 550
7. Checking client for allergies. In which are they allergic to? SATA
a. Iodine
b. Adhessive tape
c. Medication
d. Cleaning solotion
e. Latex
f. None of the above
Rationale: ABCDE – Davis’s NCLEX-RN Success Second Edition 546
8. What are examples of anti-anxiety drugs that you give during preoperative care?
a. Morphine
b. Atrophine
c. Glycopyrrolate
d. Midazolam
Rationale: D – Davis’s NCLEX-RN Success Second Edition 551
9. Which do you not observe when having a side effect on narcotics?
a. Vomiting
b. Arrhythmias
c. Hypertension
d. Respiratory depression
Rationale: C – Davis’s NCLEX-RN Success Second Edition 550
10. What are the goals of a nurse during the perioperative stage? SATA
a. Reduce Anxiety
b. Exercise to complication
c. Reduce the number of bacteria
d. Reduce the risk of aspiration during anaesthesia
e. Promote rest
f. Ensure final preparation for surgery
Rationale: ABCDEF – Davis’s NCLEX-RN Success Second Edition 551
NGT
1. What is the position of the patient when inserting the NGT?
a. Low Fowler’s position
b. Semi-Fowler’s position
c. Trendelenberg position
d. High Fowler’s position
Rationale: D – Nurse Quick Check Skills 320
2. How many mL do you irrigate when administering drugs to a client with NGT?
a. 20 ml
b. 15ml
c. 30ml
d. 25ml
Rationale: C – Nurse Quick Check Skills 318
3. How do you measure the length of the NGT before inserting it to the patient?
a. From the earlobe to the xiphoid process to the tip of the nose
b. From the xiphoid process to the tip of the nose to the earlobe
c. From the tip of the nose to the xiphoid process to the earlobe
d. From the tip of the nose to the earlobe to the xiphoid process
Rationale: D – Nurse Quick Check Skills 320
4. What is the initial thing to do when administering a drug to a client with NGT?
a. Wash your hands
b. Put on gloves
c. Verify the order
d. Explain the procedure the client
Rationale: C – Nurse Quick Check Skills 318
5. What is the important thing that you should advice a client when inserting the NGT?
a. Don’t move
b. Hold your breathe
c. Swallow
d. Cough
Rationale: C – Nurse Quick Check Skills 320
6. To minimize esophageal reflux what is the recommended position of the client?
a. Flat in bed with head slightly elevated
b. Right lateral position with head slightly elevated
c. Left lateral position with head slightly elevated
d. High fowler’s position
Rationale: B – Nurse Quick Check Skills 319
7. What is the best way to verify the position of the NGT?
a. Auscultating air
b. Aspirate gastric content
c. X-ray
d. Place the end of the tube in the container with water.
Rationale: C – Nurse Quick Check Skills 321
8. How many ml of water do you flush the NGT after administering a drug?
a. 30 to 50 ml
b. 10 to 20 ml
c. 60 to 80 ml
d. 90 to 100 ml
Rationale: A – Nurse Quick Check Skills 318
9. What is the measurement of the NGT to a child or to an infant?
a. 14 to 18 French
b. 5 to 10 French
c. 21 to 25 French
d. 11 to 13 French
Rationale: B – Delmar’s Fundamentals and Advance Nursing Skills Second Edition 715
10. Inserting and maintaining a nasogastric tube is the responsibility of a:
a. LPN
b. Physician
c. Nurse
d. Orderly
Rationale: C – Delmar’s Fundamentals and Advance Nursing Skills Second Edition 715

Colostomy

1. Most disposable pouching system in a colostomy can be used for how many days?
a. 10 days
b. 8 days
c. 9 days
d. 7 days
Rationale: D – Nurse Quick Check Skills 126
2. In how many weeks can the elimination pattern be predicted?
a. 1 – 3 weeks
b. 4 – 6 weeks
c. 7 – 9 weeks
d. 10 – 12 weeks
Rationale: B – Nurse Quick Check Skills 126
3. What kind of diet do you recommend to a client before undergoing colostomy?
a. Low residue diet
b. High protein diet
c. Low fat diet
d. High carbohydrates diet
Rationale: A – Saunders Third Edition 686
4. What kind of stool do you expect to a client with Ascending Colon Colostomy?
a. Close to normal stool
b. Loose to semiformed stool
c. Liquid stool
d. Closed to semiformed stool
Rationale: A – Saunders Third Edition 686
5. You have just assessed the client and his stoma. Which would your require to notify the physician about the client’s stoma?
a. Pink, bright red and shiny
b. Pale pink
c. Bluish
d. Purple-Black
Rationale: D – Saunders Third Edition 686
6. To which person should the irrigation and cleaning of the stoma should be delegated?
a. Ancillary Personal
b. Physician
c. Nurse
d. LVN
Rationale: C – Delmar’s Fundamentals and Advance Nursing Skills 869
7. When is the preferable time to irrigate the colostomy?
a. 2 hours after meals
b. 1 hour after meals
c. 1 hour before meals
d. 2 hours before meals
Rationale: A – Nurse Quick Check Skills 126
8. If client is ambulatory and you are about to irrigate the colostomy of the client. What is the preferred position of the client in the bathroom?
a. Standing facing the toilet
b. Sitting facing the toilet
c. Sitting underneath the shower
d. Standing by the door
Rationale: B – Nurse Quick Check Skills 126
9. How many mL is required in irrigating the colostomy?
a. 100 mL
b. 300 mL
c. 400 mL
d. 500 mL
Rationale: D – Saunders Third Edition 687
10. What is the special consideration if the client has a colostomy?
a. If the bowel continues to move between irrigations, increase the volume of irrigant.
b. No need to keep a record of the result.
c. A patient’ with an ascending or transverse colostomy won’t benefit from the irrigation
d. Don’t irrigate every other day
Rationale: C – Nurse Quick Check Skills 126

Colostomy
1. A 54 year old client underwent colon resection or Colostomy. Which nursing action is incorrect to prevent Paralytic Ileus?
a. Initiate or continue Nasogastric intubation as prescribed
b. Prepare patient for x-ray
c. Ensure adequate fluid and electrolyte replacement
d. Withhold all medications
Answer: D – Administer prescribed antibiotics id patient has symptoms of peritonitis --- Brunner & Suddarth’s Medical Surgical Nursing 10th Ed. Pg.1063
2. In Colostomy Irrigation, how high will you hang the irrigating reservoir?
a. 12-14 inches
b. 14-16 inches
c. 16-18 inches
d. 18-20 inches
Answer: D – Brunner & Suddarth’s Medical Surgical Nursing 10th Ed. Pg.1064
3. A relative of a 60 year old client ask the nurse when will the colostomy of his father begin to function. The nurse knows that a colostomy begins to function:
a. 24 hours after surgery
b. 2-3 days after surgery
c. 2-5 days after surgery
d. 3-6 days after surgery
Answer: D – Brunner & Suddarth’s Medical Surgical Nursing 10th Ed. Pg.1062
4. This kind of colostomy consists of two edges of bowel brought out onto the abdomen but separated from each other:
a. Divided Colostomy
b. Double-Barreled Colostomy
c. End or Terminal Colostomy
d. Loop Colostomy
Answer: A – The divided colostomy is often used in situations where spillage of feces into the distal end of the bowel needs to be avoided --- Fundamentals of Nursing 7th Ed. Pg.1233
5. While assessing the stoma, the nurse know that the pouch opening should be:
a. 1/8 inches smaller than the stoma
b. 1/8 inches larger than the stoma
c. 1/4 inches smaller than the stoma
d. 1/4 inches larger than the stoma
Answer: B – The stoma is measured to determine the correct size for the pouch; the pouch opening should be about 0.3 cm (1/8 in) larger than the stoma --- Brunner & Suddarth’s Medical Surgical Nursing 10th Ed. Pg.1063
6. While teaching the family of the client about colostomy care at home, what will the nurse include in her teaching? SATA
a. Inform the client of signs to report to a health care provider
b. Provide education regarding care of the ostomy and appliance when traveling
c. Educate client and family about infection control precautions
d. A visit with someone with ostomy under different circumstances maybe helpful ---similar circumstances
Answer: B – Fundamentals of Nursing 7th Ed. Pg.1250
7. A nurse knows that a bowel diversion ostomy (colostomy) are classified according to: SATA
a. Their status as permanent or temporary
b. The anatomic location
c. To divert and drain fecal material ---purpose of bowel diversion ostomy
d. The construction of the stoma
Answer: ABD – Fundamentals of Nursing 7th Ed. Pg.1231
8. While assessing an established colostomy, the nurse reports it as an unusual finding if: SATA
a. The stoma extends ½ in. above the abdomen
b. The skin under the appliance looks red briefly after removing the appliance
c. The stoma color is a deep red-purple ---red, similar in color to the mucosal lining of the inner cheek
d. An descending colostomy delivers liquid feces ---ascending colostomy
Answer: CD – Fundamentals of Nursing 7th Ed. Pg.1232, 1246
9. Which position will you put an ambulatory 45 year old client for colostomy irrigation:
a. Side lying position
b. Prone position
c. Sitting on the toilet
d. Standing by the toilet
Answer: C – If ambulatory, position the client sitting on the toilet. If on bed rest, position the client on his or her side --- Saunders 4th Edition Pg.756
10. While performing colostomy care to a 57 year old client, a nurse knows that she should empty the pouch of the client if it is:
a. 1/2 full
b. 1/3 full
c. 1/8 full
d. 2/3 full
Answer: B – Empty the pouch when it is one-third full --- Saunders 4th Edition Pg.756
11. If the pouch system id not in place, which of the following can you apply to keep the stoma moist?
a. Gauze with Petroleum Jelly
b. Gauze with NS
c. Gauze with Alcohol
d. Gauze with Hydrogen Peroxide
Answer: A – Apply petroleum jelly gauze over the stoma to keep it moist --- Saunders 4th Edition Pg.650
12. A client will undergo colostomy, he is instructed to eat a low-residue diet for:
a. 24-32 hours before surgery
b. 24-32 hours after surgery
c. 24-48 hours before surgery
d. 24-48 hours after surgery
Answer: C – Instruct the client to eat a low-residue diet for 1-2 days (24-48 hours) BEFORE surgery --- Saunders 4th Edition Pg.650
13. A 20 year old client underwent creation of colostomy. Which nursing diagnosis should the nurse include in the plan of care?
a. Adjustment, Impaired
b. Body Image, Disturbed
c. Denial, Ineffective
d. Nutrition: Imbalanced, less than body requirements
Answer: B – Saunders 4th Edition Pg.650-651, 771
14. What will the nurse observe to a client who has stoma prolapse?
a. Red-purple colored stoma ---Ischemia of the stoma
b. Narrowed stoma ---Stenosed
c. Sunken stoma ---Stoma Retraction
d. Protruding stoma
Answer: D – A prolapsed stoma is one in which the bowel protrudes through the stoma --- Saunders 4th Edition Pg.756, 773
15. The nurse is performing colostomy irrigation to a 52 year old client. During the irrigation, the client complains of abdominal cramping. Which of the following action should a nurse do? SATA
a. Notify the physician
b. Clamp of the tubing
c. Give medication for pain
d. Stop the irrigation temporarily
Answer: BD – Allow tepid fluid to enter the colon slowly. If cramping occurs, clamp off the tubing and allow the patient to rest before progressing --- Brunner & Suddarth’s Medical Surgical Nursing 10th Ed. Pg.1064
16. While inserting the catheter for colostomy irrigation, the nurse knows that it should be:
a. 7cm
b. 9cm
c. 12cm
d. 14cm
Answer: A – Insert the catheter NO MORE THAN 8cm (3 in) --- Brunner & Suddarth’s Medical Surgical Nursing 10th Ed. Pg.1064
17. In teaching the client about how to reduce odor in the ostomy drainage bag, the nurse knows that the following food will reduce the odor: SATA
a. Buttermilk
b. Beet Greens
c. Yogurt
d. Parsley
Answer: ALL – Spinach also reduces odor but is a gas-forming food as well. Broccoli, cucumbers, and eggs are gas-forming foods --- Saunders 4th Edition Pg.756, 773-774
18. In delegation, the care of a new ostomy is not delegated to a:
a. Registered Nurse
b. Nurse Practitioner
c. LPN
d. UAP
Answer: D – Care of a new ostomy is not delegated to UAP. However, aspects of ostomy function are observed during usual care and may be recorded by persons other than the nurse. Abnormal findings must be validated and interpreted by the nurse --- Fundamentals of Nursing 7th Ed. Pg.1246
19. While teaching the client about colostomy care, the nurse can instruct the client to put it on the peristomal skin if irritation occurs:
a. Half-strength Hydrogen Peroxide
b. Nystatin Powder
c. Moisten gauze
d. Alcohol
Answer: B – Brunner & Suddarth’s Medical Surgical Nursing 10th Ed. Pg.1062-1603
20. A 67 year old client is at risk of developing Mechanical Obstruction. Which interventions should a nurse perform?
a. Prepare for chest x-ray
b. Administer antibiotic as prescribed
c. Evaluate client for nausea, hiccups, chills, spiking fever, tachycardia
d. Assess client for intermittent colicky pain, nausea, and vomiting
Answer: D – Brunner & Suddarth’s Medical Surgical Nursing 10th Ed. Pg.1063
NGT
1. A 60 year old client with a Nasogastric tube (NGT) was given a bolus feeding. What position will the nurse place the client after feeding?
a. Side Lying
b. Semi Fowler’s
c. High Fowler’s
d. Prone
2. How long will the nurse maintain the position of the client in the previous question?
a. 15 minutes
b. 30 minutes
c. 1 hour
d. 1 1/5 hour
Answer: C & A – For bolus feeding, maintain the client on high Fowler’s position for 30 minutes after the feeding --- Saunders 4th Edition Pg.252
3. A 70 year old client with a Nasogastric tube (NGT) was given a continuous feeding. What position will the nurse place the client at all times?
a. Side Lying
b. Semi Fowler’s
c. High Fowler’s
d. Prone
Answer: B –Saunders 4th Edition Pg.252
4. A client asks the nurse what is the purpose of having a Nasogastric tube. The nurse knows that the purpose of NGT is: prioritize
a. To irrigate the stomach
b. To decrease the risk for aspiration
c. To provide nutrition
d. To decompress the stomach
5. Order the above according to prioritization:
Answer: DCBA – Saunders 4th Edition Pg.252
6. While assessing the client’s double-lumen NGT the nurse noticed a leakage through the air vent. What should the nurse initially do?
a. Clamp the lumen
b. Call the physician
c. Irrigate the main lumen with tap water
d. Instill 30 ml of water into the air vent
Answer: D – Saunders 4th Edition Pg.252
7. Which of the following action/s is/are correct in NGT irrigation: SATA
a. Instill 20-30mL of water or NS with an irrigation syringe
b. Instill 30-50mL of water or NS with an irrigation syringe
c. Perform irrigation every 8 hours to assess patency of the tube ---every 4 hours
d. Pull back on the syringe plunger to withdraw fluid, repeat if the tube flow is sluggish
Answer: BD – Saunders 4th Edition Pg.252
8. The nurse is about to give a bolus feeding to a client, what will indicate the nurse to withhold the feeding?
a. Residual amount of 100mL
b. Residual amount of 120mL
c. pH measuring 3.5
d. pH measuring 2.5
Answer: A – Saunders 4th Edition Pg.252-254
9. Which of the following actions are correct in NGT insertion to a 2 year old client: SATA
a. Place the infant in an infant seat or position the client with rolled towel or pillow under the head and shoulders
b. Do not hyperextend or hyper flex the patients neck
c. Restrain can be use during tube insertion
d. Tape the tube to the area between the end of the nares and upper lip as well as to the cheek
Answer: ALL – Fundamentals of Nursing 7th Ed. Pg.1209
10. In NGT insertion, to determine the accurate measurement of the length of the tube the nurse should:
a. Place the tube at the tip of the nose and measure by extending to the earlobe and then down the sternum
b. Place the tube at the earlobe and measure by extending to the tip of the nose and then down the sternum
c. Place the tube at the tip of the nose and measure by extending to the earlobe and then down the xiphoid process
d. Place the tube at the earlobe and measure by extending to the tip of the nose and then down the xiphoid process
Answer: C – Saunders 4th Edition Pg.254
11. In NGT insertion, the neck should be:
a. Extended
b. Hyperextended
c. On the side
d. Neutral position
Answer: B – Hyperextension of the neck reduces the curvature of the nasopharyngeal junction --- Fundamentals of Nursing 7th Ed. Pg.1206
12. While inserting a Nasogastric tube to an adult client, the client begins to have difficulty of breathing. What is the most appropriate nursing action?
a. Notify the physician
b. Continue to insert the tube
c. Remove the tube and reinsert immediately
d. Immediately withdraw and wait until DOB subsides
Answer: D – Saunders 4th Edition Pg.254
13. A prioritize nursing diagnosis to a client receiving tube feeding is:
a. Risk for deficient fluid volume related to hypertonic dehydration
b. Deficient knowledge about home tube feeding regimen
c. Imbalanced Nutrition: Less than body requirements
d. Risk for diarrhea related to dumping syndrome
Answer: C – Brunner & Suddarth’s Medical Surgical Nursing 10th Ed. Pg.994
14. Which of the following action/s is/are incorrect in providing oral and nasal hygiene to a client with NGT: SATA
a. Frequent mouth care
b. Clean the nose with moistened cotton-tipped swabs
c. Cleansing the nose using water soluble lubricant
d. Changing nasal tapes every 3-5 days
Answer: D – Brunner & Suddarth’s Medical Surgical Nursing 10th Ed. Pg.991
15. In giving medication via Nasogastric tube, which of the following actions should you prioritize/do first:
a. Flush with 30-50 mL of water or NS
b. Check placement and residual amount
c. Ensure that the medication can be crush
d. Crush medications or use elixir forms of medications
e. Clamp the tube for 30-60 minutes
f. Draw up the medication into the catheter tip syringe
g. Dissolve crushed medication in 5-10 mL of water
h. Insert the medication into the tube
16. Order the above step-by-step:
Answer: CDGBFHAE – Saunders 4th Edition Pg.254
17. In removing the NGT, the nurse should instruct the client to do which of the following action?
a. Exhale
b. Inhale then hold breath
c. Exhale and hold breath
d. Inhale and exhale quickly
Answer: C – Fundamentals of Nursing 7th Ed. Pg.1208
18. Which of the following actions are incorrect in removing a Nasogastric tube: SATA
a. Put on sterile gloves ---disposable
b. Pinch the tube with the gloved hand
c. Slowly and smoothly withdraw the tube --- quickly
d. Observe the intactness of the tube
Answer: A & C– Fundamentals of Nursing 7th Ed. Pg.1208
19. Before removing, the nurse may intermittently clamp and unclamp the NG tube for a trial period to ensure that the client does not experience N/V, or distention. How long is the trial period?
a. 12 hours
b. 24 hours
c. 32 hours
d. 48 hours
Answer: B – Brunner & Suddarth’s Medical Surgical Nursing 10th Ed. Pg.991
20. In removing the NG tube, the tube is gently withdrawn for:
a. 5-10 cm
b. 10-15 cm
c. 15-20 cm
d. 20-25 cm
Answer: C – Brunner & Suddarth’s Medical Surgical Nursing 10th Ed. Pg.991

Perioperative Nursing Care
1. The nurse instruct a 72 year old client who will undergo colostomy to eat:
a. Low fiber diet for 2-3 days after surgery
b. Low residue diet for 1-2 days before surgery
c. Low potassium diet for 3-6 days before surgery
d. High sodium diet for 2-4 days after surgery
Answer: B – Instruct the client to eat a low-residue diet for 1-2 days (24-48 hours) BEFORE surgery --- Saunders 4th Edition Pg.650
2. The nurse is teaching the client about Deep-Breathing and Coughing exercise before the surgery. Which of the following instructions are incorrect? SATA
a. Instruct the client that a sitting position gives the best lung expansion for coughing and deep-breathing exercises
b. Instruct the client to breathe deeply four times, inhaling through the nostrils and exhaling slowly through pursed lips ---three times
c. Instruct the client that the fourth breath should be held for 5 seconds; then, the client should cough deeply three times ---third breath, 3 seconds
d. The client should perform this exercise every 2-4 hours ---1-2 hours
Answer: BCD – Saunders 4th Edition Pg.225
3. Which of the following instruction should the nurse give first to a 34 year old client about incentive spirometry?
a. Instruct the client to repeat this process 10 times every hour
b. Instruct the client to inhale slowly to raise and maintain the flow rate indicator between the 600 and 900 marks
c. Instruct the client to place the mouth tightly around the mouthpiece
d. Instruct the client to hold the breath for 5 seconds and then to exhale through pursed lips
e. Instruct the client to assume a sitting or upright position
4. Order the above step-by-step:
Answer: ECBDA – Saunders 4th Edition Pg.225
5. How long should a nurse withhold fluid and foods for a client undergoing general anesthesia?
a. 1-2 hours
b. 3 hours ---for local anesthesia
c. 3-6 hours
d. 6-8 hours
Answer: D – Saunders 4th Edition Pg.224
6. In teaching leg and foot exercises, how should the nurse instruct a client to perform Calf Pumping?
a. Instruct the client to move both ankles by pointing the toes up and then down
b. Instruct the client to press the back of the knees against the bed, and then relax the knees; this contracts and relaxes the thigh and calf muscle to prevent thrombus formation ---Quadriceps(thigh) setting
c. Instruct the client to rotate each foot in a circle ---Foot Circles
d. Instruct the client to flex the knee and thigh and to straighten the leg and hold the position for 5 seconds before lowering ---Hip and Knee Movements
Answer: A – Saunders 4th Edition Pg.225
7. A 31 year old client asks a nurse the importance of increasing the protein intake for wound healing. The nurse should know that increase protein need is to:
a. To restore normal weight --- Calories
b. Enhances resistance to infection ---Vitamin A
c. To replace the lean body mass lost during the catabolic phase after stress
d. Important for capillary formation, tissue synthesis, and wound healing through collagen formation ---Vitamin C
Answer: C – Protein is also needed to restore blood volume and plasma proteins lost through exudates, bleeding from the wound, and possible hemorrhage; to replace losses resulting from immobility; to meet the increased needs for tissue repair and resistance to infection --- Brunner & Suddarth’s Medical Surgical Nursing 10th Ed. Pg.404
8. Order the following Diaphragmatic breathing procedures:
a. Breathe out gently and fully as the ribs sink down and inward toward midline
b. Hold this breath for a count of five
c. Practice in the same position you would assume in bed after surgery; a semi-Fowler’s position, propped in bed with the back and shoulder well supported with pillows
d. Then take a deep breath through your nose and mouth, letting the abdomen rise as the lungs fill with air
e. Practice this twice a day preoperatively
f. Exhale and let out all the air through your nose and mouth
g. With your hand in a loose-fist position, allow the hands to rest lightly on the front of the lower ribs, with your fingertips against lower chest to feel the movement
h. Repeat this exercise 15 times with a short rest after each group of five
Answer: CGADBFHE – Diaphragmatic breathing refers to a flattening of the dome of the diaphragm during inspiration, with resultant enlargement of the upper abdomen as air rushes in --- Brunner & Suddarth’s Medical Surgical Nursing 10th Ed. Pg.410
9. Which local anesthesia agents are use in dentistry and are administer topical or by injection: SATA
a. Lidocaine
b. Mepivacaine
c. Procaine
d. Tetracaine
Answer: AB – Advantages are: rapid, longer duration of action and free from local irritative effect --- Brunner & Suddarth’s Medical Surgical Nursing 10th Ed. Pg.430
10. Which phases of wound healing where blood clots are forming, wound becomes edematous, and debris of damaged tissue and blood clot are phagocytosed?
a. Fibroblastic Phase ---Proliferative Phase
b. Plateau Phase ---Maturation Phase
c. Exudative Phase
d. Connective Tissue Phase ---Proliferative Phase
Answer: C – First phase is the inflammatory phase also called lag or exudative phase --- Brunner & Suddarth’s Medical Surgical Nursing 10th Ed. Pg.450
11. The nurse is teaching the client how to apply and antiemboli stocking. Which of the following procedures is incorrect?
a. Reach inside the stocking from the top, and grasping the heel, turn the upper portion of the stocking inside out so the foot portion is inside the stocking leg.
b. Ask the client to point his or her toes, then position the stocking on the client’s foot
c. Grasp the loose portion of the stocking at the ankle and gently pull the stocking over the leg, turning it right side out in the process
d. Remove the stockings for 30 minutes every 4 hours inspecting the legs and skin while the stocking are off ---30 minutes every 8 hours
Answer: D – Fundamentals of Nursing 7th Ed. Pg.908-909
12. Which nursing diagnosis should be appropriate for the intraoperative client?
a. Risk for Perioperative-Positioning Injury
b. Risk for Deficient Fluid Volume
c. Risk for Imbalanced Body Temperature
d. Risk for Aspiration
13. Order the above:
Answer: DACB – Fundamentals of Nursing 7th Ed. Pg.911
14. Which potential postoperative problems should the nurse prioritize first?
a. Pneumonia
b. Atelectasis
c. Pulmonary Embolism
d. Urinary Retention
15. Order the above according to prioritization:
Answer: ABCD – Fundamentals of Nursing 7th Ed. Pg.915
16. After MRM (Modified Radical Mastectomy), a 47 year old client is at risk of developing Atelectasis. Which of the following action should a nurse do to prevent this complication? SATA
a. Deep Breathing and Coughing Exercise
b. Sequential Compression Device
c. Early ambulation
d. Moving in bed
Answer: ACD – Fundamentals of Nursing 7th Ed. Pg.915
17. The nurse is teaching the family of the client about suture removal at home. Which teaching is correct? SATA
a. Perform the procedure in a well-lighted, private room area of the home
b. Instruct the client to observe the incision daily and call the health care provider if increased redness, drainage, or open areas are observed
c. Provide instructions and supplies for care of the incision, and tell the client when to shower for the first time
d. Assess the client’s ability to keep the incision clean and protected at home
Answer: ALL – Fundamentals of Nursing 7th Ed. Pg.932
18. In Delegating, which medical personnel can reinforce teaching, and assist the client with deep breathing and leg exercises?
a. Registered Nurse
b. Nurse Practitioner
c. LPN/LVN
d. UAP
Answer: D – Preoperative teaching is done by the nurse and not delegated to the UAP. The UAP, however, can reinforce teaching, and assist the client with exercises, and report to the nurse if the client is unable to perform the exercises --- Fundamentals of Nursing 7th Ed. Pg.903
19. General anesthesia consists of four stages. Which stage has manifestations of ringing, roaring, or buzzing in the ears and, though still conscious, may sense an inability to move the extremities easily?
a. Beginning Anesthesia
b. Excitement
c. Surgical Anesthesia
d. Medullary Depression
Answer: A – Brunner & Suddarth’s Medical Surgical Nursing 10th Ed. Pg.423-424
20. Which postoperative discharge teaching is incorrect? SATA
a. Instruct the client to cover incision with towel if showering is allowed ---plastic
b. Be sure the client is provided with a 48 hour supply of dressing for home use
c. Instruct the client on the importance of returning to the physician’s office 7-10 days after surgery
d. Instruct client to avoid lifting for 8 weeks if a major surgical procedure was preformed ---6 weeks
Answer: AD – Saunders 4th Edition Pg.233
Tracheostomy
1. The nurse is teaching the client about the proper tracheostomy care at home except:
a. For tracheostomies older than 2 months, clean technique can be used for tracheostomy care
b. Tap water can be used for rinsing the inner cannula
c. Stress the importance of good hand washing technique
d. Inform the client/relatives of the signs and symptoms that may indicate an infection to the stoma site or lower airway
Answer: A - You can use clean technique as early as 1 month --- Fundamentals of Nursing 7th Ed. Pg.1318
2. Prioritize the purpose of Tracheostomy Care:
a. Promote comfort
b. Prevent infection
c. Facilitate healing
d. Maintain airway patency
3. Order the above according to prioritization:
Answer: DBCA - The first purpose in providing tracheostomy care is to maintain airway patency --- Fundamentals of Nursing 7th Ed. Pg.1315
4. A complication of Tracheostomy in which the tube is malpositioned causing its distal tip to push against the lateral wall of the tracheostomy:
a. Tracheoesophageal fistula
b. Tracheal Stenosis
c. Trachea-innominate artery fistula
d. Tracheomalacia
Answer: C - Saunders 4th Edition pg.260
5. In the above choices, which complication is life-threatening?
Answer: C - In Trachea-innominate fistula, there is heavy bleeding from the stoma --- Saunders 4th Edition pg.260
6. A client is allowed to eat with a tracheostomy. What should the nurse do first?
a. Place the client in fowler’s position
b. Place the client in sitting position
c. Ensure that the cuff is deflated, if the tube is not capped
d. Ensure that the cuff is inflated, if the tube is capped
Answer: B - The client is place on a sitting position and the cuff should be INFLATED if the tube is NOT capped --- Saunders 4th Edition pg.259
7. The nurse is preparing to suction the client. Before suctioning, what should the nurse do first?
a. Secure a consent
b. Explain the procedure to the patient
c. Placing the patient on a low fowler’s position
d. Provide the client with 100% oxygen
Answer: D -Saunders 4th Edition pg.259
8. In cleaning the tracheostomy site and inner cannula this is used:
a. Normal Saline Solution
b. Half-strength hydrogen peroxide
c. Alcohol
d. Tap Water
Answer: B - Following the physician’s orders and agency policy for cleaning the tracheostomy site and inner cannula, half-strength hydrogen peroxide is used --- Saunders 4th Edition pg.259
9. After tracheotomy, what should be the best nursing diagnosis fro the patient?
a. Communication, Impaired Verbal
b. Self-Care Deficit
c. Activity Intolerance
d. Aspiration, Risk for
10. To provide means of communication for a client with a tracheostomy, the nurse should not:
a. Use Magic Slate
b. Give paper and pencil to the client
c. Keep the call light within the client’s reach
d. Talk loud and clear in front of the client
Answer: A & D - Major objectives of nursing acre are to alleviate apprehension and to provide means of COMMUNICATION --- Brunner & Suddarth’s Medical Surgical Nursing 10th Ed. Pg.612-613
11. A client with tracheostomy developed Tracheoesophageal Fistula (TEF). Manifestations are the following except:
a. Food particles are seen in the tracheal secretions
b. The client has increased coughing and choking while eating
c. Increased air in cuff is needed to achieve a seal
d. The client receive the set tidal volume on the ventilator
Answer: D - Saunders 4th Edition pg.260
12. In preventing the client in developing Tracheal Stenosis, a complication of Tracheostomy, what should the nurse prioritize first?
a. Prevent pulling of and traction on the tracheostomy tube
b. Properly secure the tube in the midline position
c. Maintain cuff pressure
d. Minimize oronasal intubation time
13. Order the above according to prioritization:
Answer: ABCD - Saunders 4th Edition pg.260
14. Which of the following is correct in managing the cuff:
a. The cuff on the tracheostomy should be deflated
b. The pressure is maintained at less than 25 cm H2O to prevent injury
c. The pressure is maintained at less than 20 cm H2O to prevent aspiration
d. Cuff should be monitored for at least 4 hours
Answer: B - The pressure is maintained at LESS THAN 25 cm H2O to prevent injury and at MORE THAN 20 cm H2O to prevent aspiration --- Brunner & Suddarth’s Medical Surgical Nursing 10th Ed. Pg.614
15. While teaching the family of the client about how to suction his tracheostomy at home, the nurse should not include which of the following procedure?
a. Encourage the client to clear airway by coughing
b. Stress importance of adequate hydration
c. Instruct on how to determine the need for suctioning
d. Sterile gloves are used when suctioning
Answer: D - Clean gloves should be used when endotracheal suctioning is performed in the home environment [ARRC, 1999] --- Fundamentals of Nursing 7th Ed. Pg.1325
16. Which of the following action/s is/ are incorrect in tracheal suctioning?
a. Insert the catheter about 5 inches
b. Insert the catheter about 12.5cm
c. Apply intermittent suction for 5-10 seconds while withdrawing the catheter
d. Allow 5-10 minutes between suctions
Answer: D - Allow 2-3 minutes between suctions to provide an opportunity for reoxygenation of the lungs---Fundamentals of Nursing 7th Ed. Pg.1324
17. A 57 year old client is heavily bleeding from the stoma. He is diagnosed of having Tracheal-innominate artery fistula. What should the nurse initially do?
a. Remove the tracheostomy tube immediately
b. Apply direct pressure to the innominate artery at the stoma site
c. Call the physician immediately
d. Prepare the client for immediate repair surgery
Answer: A - Saunders 4th Edition pg.260
18. Which of the following actions are incorrect in changing the tapes on the tracheostomy tube?
a. Place clean twill tape in position to secure the tracheostomy tube
b. Take the tape around the back of the patient’s neck and thread it through the opposite opening of the inner cannula
c. Tighten the tape until only two fingers can be comfortably inserted under it
d. Secure with a knot
Answer: B - The tape should be thread it through the opposite opening of the OUTER cannula instead of the inner cannula --- Brunner & Suddarth’s Medical Surgical Nursing 10th Ed. Pg.614
19. While changing the tapes on the tracheostomy tube, the client coughs and the tube is dislodged. The initial nursing action is:
a. Grasp the retention sutures to spread the opening
b. Call the physician
c. Cover the tracheostomy site
d. Apply pressure on the tracheostomy site
Answer: A - A dislodged tracheostomy tube is difficult to reinsert, and respiratory distress may occur --- Brunner & Suddarth’s Medical Surgical Nursing 10th Ed. Pg.614
20. The son of a client asks the nurse what is the purpose of suctioning the tracheostomy tube. The nurse should know that the main purpose of suctioning the tracheostomy tube is:
a. To prevent aspiration
b. To prevent pneumonia
c. To maintain a patent airway clearance
d. To promote respiratory function
Answer: C - Fundamentals of Nursing 7th Ed. Pg.1322

PREDICTOR 11
(RATIONALE)
Cardio 20 items:
Dysrhythmia 5 items:
1. A client in the ICU is having an episode of unstable ventricular tachycardia. The nurse should instruct the client to do which of the following, if prescribed?
A. Assume semi-fowler’s position and then cough forcefully every 3-5 seconds
B. Breath normally and exhale deeply
C. Exhale and inhale deeply
D. Inhale deeply and cough forcefully every 1- 3 seconds
Answer: D- Cough CPR sometimes is used to treat unstable ventricular tachycardia. The client is instructed by the nurse to take a deep breath and cough hard every 1 – 3 seconds. Saunder’s 3rd edition © 2006 page 788.

2. A client is admitted to the surgical intensive care unit after a small bowel resection. The ECG monitor shows atrial fibrillation. How do you interpret this pattern?
A. Ventricular rate is regular at a rate of 60 beats per minute with identifiable P waves
B. PR interval is measurable and QRS complex is not consistent
C. Irregular ventricular rate at 130 bpm, unidentifiable P wave
D. Wide, bizarre and regular QRS complex with ventricular rate that exceeds 100 bpm.
Answer: C- In atrial fibrillation the ventricular rate is irregular at 130bpm, P waves are not identifiable and the PR interval is not measurable, QRS complexes have a consistent appearance and the baseline is fibrilatory. Springhouse NCLEX-RN Review Interactive

3. A client is connected to a cardiac monitor. The nurse notes ventricular tachycardia in the ECG monitor. The client appears to be conscious and stable. What will be the appropriate action of the nurse?
A. Administer oxygen and prepare to defibrillate the client
B. Call the physician immediately
C. Call for a code and administer oxygen
D. Administer oxygen and antidysrhythmia as prescribed
Answer: D- treatment for stable client with sustained ventricular tachycardia includes the administration of oxygen and antidysrhythmia as prescribed. defibrillation is not necessary because the client is conscious and stable. Saunder’s 3rd edition © 2006 page 788.

4. Following coronary artery bypass grafting, a client begins having chest fullness and anxiety. The nurse suspects cardiac tamponade and prints a lead II ECG strip for interpretation. In looking at the strip, the change in the QRS complex that would most support her suspicion is
A. A narrowing complex
B. A widening complex
C. An amplitude increase
D. An amplitude decrease
Answer: D- fluid surrounding the heart such as in cardiac tamponade suppresses the amplitude of the QRS complex on an ECG. Springhouse NCLEX-RN Review Interactive

5. The nurse caring for a client on the telemetry unit is able to determine that the client is in sinus bradycardia by recognizing which characteristics?
A. A P wave in front of every QRS complex
B. Atrial rate of 120 bpm
C. ST elevation greater than 0.01mm
D. P wave is absent
Answer: A- Sinus bradycardia has the following characteristics: 1) P wave is normal and consistent in shape , occurring in front of every QRS complex; 2) ventricular and atrial rate less than 60 bpm; 3) PR interval that is between 0.12 to 0.20 seconds. Lippincott’ Review for NCLEX-RN © 2005 page 685

Hypertension 5 items
1. When teaching a client with newly diagnosed of hypertension about the pathophysiology of this disease, the nurse states that the arterial baroreceptors, which monitor arterial pressure, are found in the carotid sinus and aorta. Which other area should the nurse mention as the site of arterial baroreceptors?
A. Brachial artery
B. Left ventricular wall
C. Radial artery
D. Right ventricular wall
Answer: B- arterial baroreceptors are found in the ventricular wall as well as the carotid sinus and aorta. Springhouse NCLEX-RN Review Interactive.

2. An exercise program is prescribed for the client with hypertension. Which intervention would be most likely to assist the client in maintaining an exercise program?
A. Giving the client a written exercise program
B. Tailoring a program to the client’s needs and abilities
C. Reassuring the client that he or she can do the exercise program
D. Explaining the exercise program to the client’s spouse
Answer: B- tailoring or individualizing a program to the client’s lifestyle has been shown to be an effective strategy for changing health behaviors. Providing a written program, explaining the program to the client’s spouse and reassuring the client that he or she can do the program may be helpful but are not likely to promote adherence. Lippincott’ Review for NCLEX-RN © 2005 page 244

3. The nurse in the hypertension clinic is working with a 50 year old woman who has had a mildly elevated blood pressure for the last several months. The clinic follows the recommendations of the joint National Committee on Detection, Evaluation and Treatment of high blood pressure. Which of the following studies would not be ordered for this client?
A. ECG
B. Urinalysis
C. Serum calcium levels
D. White blood cell count
Answer: D- white blood cell count is not necessary for the evaluation of a client with hypertension. The JNC recommends a complete urinalysis, hemoglobin and hematocrit, serum potassium and calcium, creatinine, and plasma cholesterol levels.

4. A home health nurse is developing a teaching plan for a client with mild hypertension. Which of the following goals is the most important for the nurse to include?
A. Control of the disease
B. Restoration of prior state of health
C. Avoidance of renal complication
D. A healthier lifestyle
Answer: A- major goal for the treatment of HPN is to control the disease/reduce the BP; secondary goals are: 1) lessen the extent of organ damage; 2) adhering to a healthier lifestyle. Saunders 3rd edition © 2006 page 809.NSNA NCLEX-RN REVIEW © 2000 page 716

5. A woman who has mild hypertension asks the nurse if she needs blood pressure medicine. The nurse explains that pharmacologic therapy is usually added to the therapeutic regime
A. Anytime a client is non-compliant with the diet therapy
B. When symptoms appear
C. When the history indicates that the client is at risk of cardiovascular disease
D. When difference between systolic and diastolic blood pressure is greater than 60 mm Hg.
Answer: C- first line drugs used in the management of hypertension are added when the client is considered to be at risk for cardiovascular disease. NSNA NCLEX-RN REVIEW © 2000 page 717

Pacemaker 5 items:
1. A client receives a pacemaker to treat a recurring arrhythmia. When monitoring the cardiac rhythm strip the nurse observes extra pacemaker spikes that aren’t followed by a beat. Which condition should the nurse suspect?
A. Failure to pace
B. Failure to capture
C. Failure to sense
D. Asystole
Answer: B- Extra pacemaker spikes that aren’t followed by a beat may indicate failure to capture in which the pacemaker fires but the heart does not conduct the beat. Springhouse NCLEX-RN Review Interactive.

2. A client is admitted to the telemetry unit for placement of a permanent pacemaker because of sinus bradycardia. A priority goal for the client with in 24 hours after insertion of permanent pacemaker would be to
A. Maintain cardiac conduction stability
B. Maintain skin integrity
C. Decrease cardiac output
D. Increase activity level
Answer: A- maintaining cardiac conduction stability to prevent dysrhythmias is a priority immediately after artificial pacemaker implantation. The client should have continuous ECG monitoring until proper pacemaker functioning is verified. Lippincott’ Review for NCLEX-RN © 2005 page 255

3. The client who had a permanent pacemaker implanted 2 days earlier is being discharged from the hospital. Which of the following indicates that the client understands the home health teachings?
A. selects a low cholesterol diet to control coronary artery disease
B. states the need for bed rest for 1 week after discharge
C. verbalizes safety precautions needed to prevent malfunction
D. explains signs and symptoms of myocardial infarction
Answer: C – the client with permanent pacemaker needs to be able to state specific information about safety precautions necessary to maintain proper pacemaker function. Lippincott’ Review for NCLEX-RN © 2005 page 255

4. After having several stoke-adams attacks over 4 months, a client reluctantly agrees to implantation of a permanent pacemaker. Before discharge the nurse reviews pacemaker care and safety guidelines with the client and spouse. Which safety precaution is appropriate for a client with permanent pacemaker?
A. Stay at least 2 feet away from microwave ovens
B. Never engage in activities that require vigorous arm and shoulder movements
C. Avoid going through airport metal detectors
D. Avoid using a cellular phone
Answer: D- a client with pacemaker should avoid using cellular phones because they may disrupt the function of the pacemaker. This problem is less likely to occur with newer microwave ovens; nevertheless the client should stay at least 5 feet away from microwave not 2 feet. The client must avoid vigorous arm and shoulder movement only for the first 6 weeks after pacemaker implantation. Metal detectors don’t harm pacemakers; however the client should notify airport security guards of the pacemaker because its metal casing and programming magnet may trigger the metal detector. Springhouse NCLEX-RN Review Interactive.

5. The nurse in the ICU is evaluating the knowledge of a newly hired nurse about pacemaker devices. Which of the following statements if made by the new nurse signifies correct understanding?
A. Permanent pacemakers does not require batteries
B. Temporary pacers are surgically implanted on the subclavian vein
C. Pacemakers are powered by lithium batteries
D. Transvenous invasive temporary pacing is used as an emergency measure when a client is being transported
Answer: C- Pacemakers are powered by a lithium battery that has a lifespan of 10 years. Nuclear powered pacers have a lifespan of 20 years or longer. Saunders 3rd edition © 2006 page 791

CAD/Valvular Disease 5 items:
1. A client comes to the ER with complains of chest pain, dyspnea and an irregular heartbeat. An ECG shows a heart rate of 110 bpm (sinus tachycardia) with frequent PVC. Shortly after admission, the client has ventricular tachycardia and becomes unresponsive; the client is taken to the ICU. Which nursing diagnosis is appropriate at this time?
A. Knowledge deficits related to interventions used to treat acute illness
B. Impaired physical mobility related to complete bed rest
C. Social isolation related to limited visiting hours in the ICU
D. Anxiety related to the threat of death
Answer: D- anxiety related to the threat of death is an appropriate nursing diagnosis at this time because anxiety can adversely affect the heart rate and rhythm by stimulating the autonomic nervous system. Also because the client required resuscitation, the threat of death is real and immediate concern. Springhouse NCLEX-RN Review Interactive.

2. While caring for a client who has sustained an MI. The nurse notes eight PVCs in 1 minute on the cardiac monitor. The client is receiving an intravenous infusion of D5 Water and oxygen at 2 liters per minute. The nurse’s first course of action should be to:
A. Increase the IV infusion rate
B. Notify the physician promptly
C. Increase the oxygen concentration
D. Administer a prescribed analgesic
Answer: B- the physician should be notified immediately if PVC occurs at a rate greater than five or six per minute in a post MI client. More than six PVCs per minute is considered serious and usually calls for decreasing ventricular irritability by administering medications such as lidocaine hydrochloride. Lippincott’ Review for NCLEX-RN © 2005 page 249
3. The nurse is assessing the client for heart failure. to assess for hepatojugular reflux the nurse should
A. Press the client’s right upper abdomen
B. Elevate the client’s head to 90 degrees
C. Press the left upper abdomen
D. Lie the client flat in bed
Answer: A- hepatojugular reflux is a sign of right sided heart failure, is assessed with the head of the bed at a 4 5 degree angle. As the right upper abdomen (area over the liver) is compressed 30 to 40 seconds, the nurse observes internal jugular vein. If the client’s internal jugular vein becomes distended, the client has positive hepatojugular reflux. Springhouse NCLEX-RN Review Interactive.


4. To check for arterial insufficiency when a client is in supine position, the nurse should elevate the extremity at a 45 degree angle and then have the client sit up. The nurse suspects arterial insufficiency if the assessment reveals which of the following?
A. Elevational rubor
B. Dependent pallor
C. No rubor for 10 seconds after the maneuver
D. 30 seconds filling time for the veins
Answer: B- if arterial insufficiency is present, elevation of the limb would yield pallor from the lack of circulation. Rubor and increase venous filling time would suggest venous problems secondary to venous trapping and incompetent valves. Springhouse NCLEX-RN Review Interactive

5. A client with massive myocardial infarction rapidly develops cardiogenic shock. Ideally the physician would use the Intra-Aortic Balloon pump to support the injured the myocardium. However the client has a history of unstable angina pectoris, aortic insufficiency, and hypertension and diabetis mellitus. Which of these conditions contraindicates the use of IAB?
A. Unstable angina pectoris
B. Aortic insufficiency
C. Hypertension
D. Diabetes mellitus
Answer: B- aortic insufficiency contraindicates the use of IABP. Other contraindications for this therapy include aortic aneurism, central or peripheral atherosclerosis, chronic end-stage heart disease, multisystemic failure, chronic debilitating disease, bleeding disorders and a history of emboli. Springhouse NCLEX-RN Review Interactive

ENDO 20 items
DM 5 Items:
1. A nurse is caring for a client with long standing diabetes mellitus. Which finding is most consistent of with damage to the autonomic nervous system?
A. Flushed, warm extremities
B. Dry, cracked skin
C. Absent pulses
D. Burning sensation on the soles of the feet
Answer: B- the blood vessels of the skin constrict in response to impulses from autonomic nervous system. This can result in dry, cracked skin when vascular constriction is of a long –standing nature. NSNA NCLEX-RN REVIEW © 2000 page 719.

2. A client comes to the emergency room with diabetic ketoacidosis. The nurse would identify which of the following?
A. Disturbed sleep pattern
B. Impaired maintenance
C. Imbalance nutrition: less than body requirements
D. Deficient fluid volume
Answer: D-deficient fluid volume causing dehydration and possible hypovolemic shock is the main problem in DKA because increased osmolarity from the glucose leads to a fluid shift from the intracellular to the extracellular space. The fluid shift leads to increased renal excretion of glucose and fluid. Lippincott’ Review for NCLEX-RN © 2005 page 372

3. A client with type I DM has been on a regimen of multiple daily injection therapy. He’s been converted to continuous subcutaneous insulin therapy. While teaching the client about continuous subcutaneous insulin therapy, the nurse would be accurate in telling the client that the regimen includes the use of:
A. Intermediate and long acting insulin
B. Short and intermediate acting insulin
C. Intermediate insulin only
D. Short acting insulin only
Answer: D- continuous subcutaneous insulin therapy uses a basal rate and boluses of short acting insulin. Multiple daily injections of insulin use a combination of short-acting and intermediate or long-acting insulins. Springhouse NCLEX-RN Review Interactive

4. Which of the following statements about fluid replacement is accurate for a client with hyperosmolar hyperglycemic nonketotic syndrome (HHNS)?
A. Administer 6 liters of I.V. fluids for the first 24 hours
B. Administer 2-3 liters of IV fluids rapidly
C. Administer D5 normal saline solution
D. Administer IV fluids slowly to prevent circulatory overload and collapse
Answer: B- regardless of the client’s medical history, rapid fluid resuscitation is critical for maintaining cardiovascular integrity. Profound intravascular depletion requires aggressive fluid replacement. A typical fluid resuscitation protocol is 6 liters over the first 12 hours, with more fluid to follow over the next 24 hours. Springhouse NCLEX-RN Review Interactive

5. A client with diabetic mellitus is prescribed with angiotensin converting enzyme inhibitor to reduce vascular changes and possibly prevent or delay development of which of the following?
A. Chronic obstructive pulmonary disease
B. Pancreatic cancer
C. Renal failure
D. Cerebro-vascular accident
Answer: C- renal failure frequently results from the vascular changes associated with DM. ACE inhibitors increase renal blood flow and are effective in decreasing diabetic nephropathy. Lippincott’ Review for NCLEX-RN © 2005 page 360

Addison’s 5 items:
1. A client with addison’s disease is admitted to the medical unit. The nurse diagnoses the client with deficient fluid volume related to inadequate fluid intake and fluid loss secondary to inadequate adrenal hormone secretion. As the client’s oral intake increases, which of the following fluids would be most appropriate?
A. Milk and diet soda
B. Water and eggnog
C. Bouillon and juice
D. Coffee and milkshakes
Answer: C- electrolyte imbalances associated with addison’s disease include hypoglycemia, hyponatremia, and hyperkalemia. Salted bouillon and fruit juices provide glucose and sodium to replenish these deficits. Diet soda does not contain sugar. Water could cause further dilution. Coffee’s diuretic effect would aggravate the fluid deficit. Milk contains potassium and sodium. Lippincott’ Review for NCLEX-RN © 2005 page 372.

2. The nurse is caring for a client in acute addisonian crisis. Which laboratory finding would the nurse expect to find?
A. Hyperkalemia
B. Reduced BUN
C. Hypernatremia
D. Hyperglycemia
Answer :A- In adrenal insufficiency, the client has hyperkalemia due to reduced aldesterone secretion. BUN increases as the glumerular filtration rate is reduced. Hyponatremia is caused by reduced aldosterone secretion. Reduced cortisol secretion leads to impaired gluconeogenesis and a reduction of glycogen in the liver and muscle causing hypoglycemia. Springhouse NCLEX-RN Review Interactive

3. An adult is readmitted to the medical/surgical care unit in addisonian crisis. He is exhibiting signs of tachycardia, hyponatremia, hyperkalemia, dehydration and hypoglycemia. The nurse would expect that the initial orders for this client would include
A. Administration of oxygen via 100% nonrebreather mask
B. Starting an IV solution of saline and dextrose
C. Administering potassium chloride
D. Preparing for an emergency tracheostomy
Answer: B- management of addisonian crisis includes glucocorticoid management and intravenous replacement of sodium and dextrose as necessary. NSNA NCLEX-RN REVIEW © 2000 page 847

4. A nurse is developing a care plan for a newly diagnosed addison’s disease. Which of the following nursing diagnosis is appropriate for the nurse to include in the plan of care?
A. Risk for infection
B. Fluid volume excess
C. Hypothermia
D. Urinary retention
Answer: A - Addison’s disease decreases the production of all adrenal hormones, compromising the bodies normal stress response and increasing the risk for infection. Other nursing diagnoses for a client with addison’s disease include fluid volume deficit and hyperthermia. Springhouse NCLEX-RN Review Interactive

5. The nurse is conducting discharge education with a client newly diagnosed with addison’s disease. Which information should the nurse include in the client and family teaching plan? SATA
A. Addison’s disease will resolve over a few weeks requiring no further treatments
B. Avoiding stress and maintaining a balanced lifestyle will minimize risk for exacerbations
C. Fatigue, weakness, dizziness and mood changes need to be reported to the physician
D. A medic alert bracelet should be worn
E. Family members need to be informed about the warning signals of adrenal crisis
F. Dental work or surgery will require adjustment of daily medications
Answer: BCDEF- addison’s disease occurs when the client does not produce enough steroids from the adrenal cortex. Lifetime steroid treatment is needed. The client should be taught lifestyle management to avoid stress and maintain rest periods. A bracelet should be worn and the family should be taught symptoms that would indicate impending adrenal crisis such as fatigue, dizziness, weakness or mood changes. Dental work or surgery will often require an adjusted dosage of steroids. Lippincott’ Review for NCLEX-RN © 2005 page 373.

SIADH 5 items:
1. A client is admitted with syndrome of inappropriate antidiuretic hormone (SIADH). Which laboratory finding should the nurse anticipate?
A. Decreased sodium
B. Decreased serum creatinine level
C. Increased hematocrit
D. Increased BUN level
Answer: A- in SIADH, the posterior pituitary gland produces excess anti-diuretic hormone (vasopressin) which decreases water excretion by the kidneys. This in turn reduces the serum sodium level, causing hyponatremia. In SIADH, the serum creatinine level isn’t affected by the client’s fluid status and remains within normal levels. Springhouse NCLEX-RN Review Interactive


2. Which of these signs suggest that the client with syndrome of inappropriate anti-diuretic hormone secretion is experiencing complications?
A. Tetanic contractions
B. Neck vein distension
C. Weight loss
D. Polyuria
Answer: B- SIADH causes anti-diuretic hormone overproduction, which leads to fluid retention. Severe SIADH can cause such complications as vascular overload, signaled by neck vein distention. This syndrome is not associated with titanic contractions. It may cause weight gain and fluid retention. Springhouse NCLEX-RN Review Interactive

3. An adult client diagnosed with SIADH asks the home health nurse about necessary diet modifications she has to observe in order to minimize complications of the disease. The nurse’s best response would be
A. “ Minimize intake of foods high in protein and carbohydrates”
B. “ Avoid foods high in potassium”
C. “ Eat foods high in calcium and sodium”
D. “ There is no need to modify your diet”
Answer: A- clients with SIADH need to modify their intake of protein, carbohydrates and sodium. The use of salt tablets if prescribed or ingestion of salty foods such as junk foods may be instructed to the client if the client is experiencing increase sweating. NSNA NCLEX-RN REVIEW © 2000 page 327

4. A nurse is developing a discharge teaching with a client diagnosed with SIADH. The following discharge instructions should be included except
A. Avoid strenuous exercise especially in hot weather
B. Recognize signs of adrenal insufficiency
C. Increase water intake during the day
D. Avoid stressful situations and alternate activities with rest periods
Answer: C- clients with SIADH should be instructed to avoid strenuous exercises during hot weather, watch out for signs of adrenal insufficiency to prevent complications, avoid stressful activities and provide rest periods in between activities and restrict fluid intake not increase because the client is susceptible to fluid overload. NSNA NCLEX-RN REVIEW © 2000 page 327

5. A client is admitted in the nursing unit for the treatment of syndrome of inappropriate antidiuretic hormone. Which nursing intervention is inappropriate?
A. Infusing IV fluids rapidly
B. Restricting fluid intake
C. Encouraging adequate rest periods
D. Administerind demeclocycline as prescribed
Answer: A- interventions for a client with SIADH includes restricting fluid intake, monitoring intake and output, providing safe environment and adequate rest periods and administering medications as prescribed by the physician. Administering fluids by any route would further increase the client’s already heightened fluid load. Saunders NCLEX-RN 3rd edition © 2006 page 632

Thyroid diseases 5 items:
1. An adult client is diagnosed with hypothyroidism. Which signs and symptoms would the nurse expect the client to experience? Select all that apply
A. Rapid pulse
B. Decreased energy and fatigue
C. Fine, thin hair with hair loss
D. Constipation
E. Weight gain of 10 lbs
F. Menorrhagia
Answer: B, C, E, F- clients with hypothyroidism exhibit symptoms indicating a lack of thyroid hormone. Bradycardia, decrease energy and lethargy, memory problems, coarse hair, constipation, weight gain and menorrhagia are common signs and symptoms of hypothyroidism. Lippincott’s NCLEX-RN Review © 2005 page 367

2. A nurse is caring for a client post thyroidectomy. The client is exhibiting hyperreflexia, muscle twitching and spasms. The first action the nurse should perform is to
A. Assess for additional signs of tetany
B. Prepare to send a blood sample to the laboratory for a calcium level
C. Place the client in semi-fowler’s position
D. Administer post –op pain medication
Answer: B- during thyroidectomy, it is possible for the parathyroid gland to be removed or damaged. If the parathyroid glands are disturbed, hypocalcemia may result. NSNA NCLEX-RN REVIEW © 2000 page 327

3. A client with hyperparathyroidism declined surgery and is to receive hormonal replacement therapy with estrogen and progesterone. Which of the following instructions is the most important for the nurse to include in the client’s teaching plan?
A. Maintain a moderate exercise program
B. Rest as much as possible
C. Lose weight
D. Jog at least 2 miles per day
Answer: A- A moderate exercise program will help strengthen bones and prevent the bone loss that occurs from excess parathyroid hormone. Walking or swimming provides the most beneficial exercise. Because of weaker bones a rigorous exercise such as jogging would be contraindicated. Weight loss might be beneficial but it isn’t as important as developing a moderate exercise program. Springhouse NCLEX-RN Review Interactive

4. Which of the following nursing diagnoses is appropriate for a client with grave’s disease performing self-care after treatment with RAI in the form of sodium iodide 131 I?
A. Risk for injury related to altered level of consciousness
B. Ineffective breathing pattern related effects of radio-active iodine
C. Total self-care deficit related to the need for immobilization after RAI therapy
D. Risk for ineffective therapeutic regimen management related to lack of knowledge about the disease
Answer: D- management of the disease process is a priority for the client who has undergone RAI therapy with sodium iodide 131 I. signs of hypothyroidism usually persists for 1 to 2 months and may still be present for up to 1 year until thyroid hormone production stops. Permanent hypothyroidism is the major complication of radioactive 131 I treatment. Lippincott’s NCLEX-RN Review © 2005 page 367

5. Which nursing diagnosis takes the highest priority for a client with hyperthyroidism?
A. Altered nutrition: Risk for more than body requirements related to thyroid hormone excess
B. Risk for impaired skin integrity related to edema, skin fragility and poor wound healing
C. Body image disturbance related to weight gain and edema
D. Altered nutrition less than body requirement related to thyroid hormone excess
Answer: D- in a client with hyperthyroidism, excessive thyroid hormone production leads to hypermetabolism and increased nutrient metabolism. This condition may result in a negative nitrogen balance, increased protein sythesis and breakdown, decreased glucose tolerance and fat mobilization and depletion. This puts the client at risk for marked nutrient and calorie deficiency, making altered nutrition less than body requirement the most important nursing diagnosis. Option B and C may be appropriate for a client with hypothyroidism, which slows down metabolic rate. Springhouse NCLEX-RN Review Interactive

Renal
Acute renal failure 5 items:
1. A client is admitted to the medical surgical unit for the treatment of acute renal failure. During routine nursing care, the nurse must continually assess for which of the following?
A. hyponatremia and hypokalemia
B. decreased BUN and creatinine
C. alkalosis
D. hypercalcemia
Answer: A- the most common findings in acute renal failure include hyponatremia, hyperkalemia, hypocalcemia and the elevation of BUN and creatinine. NSNA NCLEX-RN REVIEW © 2000 page 841

2. A client develops decreased renal function and requires a change in antibiotic dosage. On which factor would the physician base the dosage change?
A. GI absorption rate
B. Therapeutic level
C. Creatinine clearance
D. Liver function studies
Answer: C-the physician orders creatinine clearance to gauge the glumerular filtration rate of the kidneys. This is important because most drugs are excreted partially by the kidneys. Springhouse NCLEX-RN Review Interactive

3. The client with acute renal failure is recovering and asks the nurse ”will my kidneys ever function normally again?” the nurse’s response is based on the knowledge that the client’s renal status will most likely
A. Continue to improve over the period of weeks
B. Result in the need for permanent hemodialysis
C. Improve only if the client receives a renal transplant
D. Result in end-stage renal failure
Answer: A- the kidneys have a remarkable ability to recover from serious insult. Recovery may take 3- 12 months. The client must be taught how to recognize the symptoms of decreasing renal function and to notify the physician if such problems occur. In a client recovering from acute renal failure, there is no need for renal transplantation or permanent hemodialysis. Chronic renal failure develops before end-stage renal failure. Lippincott’s NCLEX-RN Review © 2005 page 381

4. A client is admitted with acute renal failure. The nurse should monitor closely for which of the following?
A. Enuresis
B. Lethargy
C. Drug toxicity
D. Insomnia
Answer: C- acute renal failure is characterized by oliguria and rapid accumulation of nitrogen waste in the blood.kidneys excrete medications so the nurse should monitor closely for drug toxicity. With decrease urinary output or no output enuresis should not occur. The client will most likely feel lethargic, but this is not as serious as drug toxicity. Springhouse NCLEX-RN Review Interactive

5. A high carbohydrate, low protein diet is prescribed for the client with acute renal failure. the rationale for this diet is that carbohydrates will
A. Act as a diuretic
B. Reduce demands on the liver
C. Help maintain urine acidity
D. Prevent the development of ketosis
Answer: D- a high carbohydrate foods meet the body’s caloric needs during acute renal failure. Protein is limited because its breakdown may result in accumulation of toxic waste products. The main goal of nutrient therapy in ARF is to decrease protein catabolism. Protein catabolism causes increased levels of urea, phosphate and potassium. Carbohydrates provide energy and decrease the need for protein breakdown. Lippincott’s NCLEX-RN Review © 2005 page 380

Hemo/peritoneal dialysis 5 items:
1. A client with acute renal failure is undergoing dialysis for the first time. The nurse monitors the client closely for dialysis equilibrium syndrome, a complication that is most common during the first few dialysis sessions. Typically dialysis equilibrium syndrome causes
A. Confusion, headache and seizure
B. Acute bone pain and confusion
C. Weakness, tingling and cardiac arrhythmias
D. Hypotension, tachycardia and tachypnea
Answer: A- dialysis equilibrium syndrome causes confusion, a decreasing level of consciousness, headache and seizures. These findings which may last several days probably result from a relative excess of interstitial or intracellular solutes caused by rapid solute removal from the blood. The resultant organ swelling interferes with normal physiologic functions. To prevent this syndrome, many dialysis centers keep first time sessions short and use a reduce blood flow rate. Springhouse NCLEX-RN Review Interactive

2. If disequilibrium syndrome occurs during dialysis, which of the following would be the priority nursing action?
A. Administer oxygen per nasal cannula
B. Slow the rate of the dialysis
C. Reassure the client that the symptoms are normal
D. Place the client in trendelenberg position
Answer: B- if disequilibrium syndrome occurs during dialysis, the most appropriate action is to slow the rate of dialysis. The syndrome is believed to be the result of too-rapid removal of urea and excess electrolytes from the blood; this causes transient cerebral edema which produces the symptoms. Lippincott’s NCLEX-RN Review © 2005 page 381

3. The nurse knows that the client on continuous ambulatory peritoneal dialysis understands his treatment when the client states,
A. “ I must increase my carbohydrate intake daily”
B. “ I must maintain a positive nitrogen balance by decreasing proteins”
C. “ I must take prophylactic antibiotics to prevent infection”
D. “ I must be aware of the signs and symptoms of peritonitis”
Answer: D- peritonitis is a life- threatening complication of CAPD which is manifested by abdominal pain and distention, diarrhea, vomiting and fever. Clients are given antibiotics orally or parenterally as necessary not prophylactically. NSNA NCLEX-RN REVIEW © 2000 page 841

4. Because of difficulties with hemodialysis, peritoneal dialysis is initiated to treat a client’s uremia. Which finding signals a significant problem during this procedure?
A. Blood glucose level of 200mg/ dl
B. White blood cell count of 20,000/mm
C. Potassium level of 3.5 mEq/L
D. Hematocrit of 35%
Answer: B- an increase WBC count indicates infectionprobably resulting from peritonitis, which may have been caused by insertion of the peritioneal catheter in to the peritoneal cavity. Peritonitis can cause the peritoneal membrane to lose its ability to filter solutes; therefore, peritoneal dialysis would no longer be a treatment option for this client. Springhouse NCLEX-RN Review Interactive

5. A client requires hemodialysis. Which of the following drugs should be withheld before the procedure?
A. Phosphate binders
B. Insulin
C. Antibiotics
D. Cardiac glycosides
Answer: D- cardiac glycosides should be withheld before hemodialysis. Hypokalemia is one of the electrolyte shifts that occur during dialysis, and hypokalemic clients are at risk of digitalis toxicity. Springhouse NCLEX-RN Review Interactive

Chronic Renal Failure 5 items:
1. The nurse assesses the client who has chronic renal failure and notes the following: crackles in the lung bases, elevated blood pressure, and weight gain of 2 lbs in a day. Based on these data, which of the following nursing diagnoses is appropriate?
A. Excess fluid volume related to the kidney’s inability to maintain fluid balance
B. Increased cardiac output related to fluid overload
C. Ineffective tissue perfusion related to interrupted arterial blood flow
D. Ineffective therapeutic regimen related to lack of knowledge about therapy
Answer: A- crackles in the lungs, weight gain and elevated blood pressure are indicators of excess fluid volume a common complication in chronic renal failure. Lippincott’s NCLEX-RN Review © 2005 page 383

2. A client with chronic renal failure has developed faulty red blood cell production. The nurse should monitor the client for
A. Nausea and vomiting
B. Dysnea and cyanosis
C. Fatigue and weakness
D. Thrush and circumoral pallor
Answer: C- decrease RBC productiondiminishes cellular oxygen leading to fatigue and weakness. Springhouse NCLEX-RN Review Interactive

3. In planning teaching strategies for a client with chronic renal failure, the nurse must keep in mind the neurologic impact of uremia. Which teaching strategy would be most appropriate?
A. Providing all needed teaching in one extended session
B. Validating frequently the client’s understanding of the material
C. Conducting one-on – one teaching with the client
D. Using videotapes to reinforce the material as needed
Answer: B- uremia causes decreased alertness, so the nurse needs to validate the client’s comprehension frequently. Because the client’s ability to concentrate is limited, short sessions are most effective. If family members are present at the sessions, they can reinforce the material. Written materials are superior to videotapes because the client may not be able to maintain alertness during the viewing of the videotapes. Lippincott’s NCLEX-RN Review © 2005 page 384

4. The nurse helps the client with chronic renal failure develop a home diet plan with the goal of helping the client maintain adequate nutritional intake. Which of the following diets would be most appropriate for a client with chronic renal failure?
A. High carbohydrate, high protein
B. High calcium, high protein, high potassium
C. Low protein, low potassium, low sodium
D. Low protein, low potassium
Answer: C- dietary management for a client with chronic renal failure is usually designed to restrict protein, sodium and potassium intake. Protein intake is reduced because the kidney can no longer excrete byproducts of protein metabolism. The degree of dietary restriction usually depends on the degree of renal impairment. The client should also receive high carbohydrate diet along with appropriate vitamin and mineral supplements. Lippincott’s NCLEX-RN Review © 2005 page 384

5. Sexual problems can be troublesome to clients with chronic renal failure. Which of the following strategies would be most useful in helping the client cope with such a problem?
A. Help the client to accept that sexual activity will be decreased
B. Suggest using alternative forms of sexual expression and intimacy
C. Tell the client to plan rest periods after sexual activity
D. Suggest that the client avoid sexual activity to prevent embarrassment
Answer: B- altered sexual functioning commonly occurs in chronic renal failure and can stress marriage and relationships. Altered sexual functioning can be caused by decreased hormone levels, anemia, peripheral neuropathy or medication. The client should not decrease or avoid sexual activity but instead should modify it. The client should rest before sexual activity. Lippincott’s NCLEX-RN Review © 2005 page 384

BPH 5 items:
1. After having transurethral resection of the prostate, a client is returned to the nursing unit with a three way indwelling urinary catheter and continuous closed bladder irrigation. Which finding suggests that the client’s catheter is occluded?
A. The urine in the drainage bag appears red to pink
B. The client reports bladder spasm and the urge to void
C. The normal saline irrigant is infusing at a rate of 50 drops/minute
D. About 1000 ml of irrigant is being instilled and 1200ml of drainage have been returned
Answer: B- reports of bladder spasm and the urge to void suggest that a blood clot may be occluding the catheter. After TURP, the drainage normally appears red to pink and normal saline irrigant is usually infused at a rate of 40 to 60 drops per minute. Springhouse NCLEX-RN Review Interactive

2. After undergoing transurethral resection of the prostate to treat benign prostatic hyperplasia, a client returns to the room with a continuous bladder irrigation. On the first day after surgery, the client reports bladder pain. What should the nurse do first?
A. Increase the IV flow rate
B. Notify the physician immediately
C. Administer meperidine (Demerol) 50 mg IM as prescribed
D. Assess the irrigation catheter for patency and drainage
Answer: D- although post operative pain is expected, the nurse should make sure that other factors, such as an obstructed irrigation catheter is not the cause of the pain. After assessing catheter patency, the nurse should administer analgesic such as meperindine as prescribed to relieve pain and discomfort. Increasing the IV flow rate may worsen the pain. Springhouse NCLEX-RN Review Interactive

3. The nurse is completing an admission assessment on a client with benign prostatic hyperplasia. The nurse should obtain an in-depth assessment about
A. Laboratory studies
B. Urinary patterns
C. Electrocardiograms
D. Internal bleeding
Answer: B-benign prostatic hyperplasia is the growth of new cells in the prostate gland resulting in urinary obstruction; therefore, assessment of the obstructive symptoms are: decrease in the force of the urinary stream, hesitancy in initiation of urine; dribbling, urinary retention, incomplete bladder emptying, nocturia, dysuria and urgency. NSNA NCLEX-RN REVIEW © 2000 page 841

4. A client with BPH is at the clinic for follow up. Which of the following statements indicate to the nurse that the client understands the management of his condition?
A. “As soon as I finish this visit, I will never have to worry about BPH again”
B. “ I don’t know how am I going to get used to voiding every 2 to 3 hours”
C. “ I will wear an athletic supporter while I am awake”
D. “ I am going to avoid fluids while at work to prevent dribbling”
Answer: B- clients with BPH should void every 2 to 3 hours to flush the urinary tract. NSNA NCLEX-RN REVIEW © 2000 page 842

5. AN elderly client with benign prostatic hyperthrophy is scheduled to have a transurethral resection of the prostate in a few hours. The client asks the nurse “how will my sex life be affected?” the nurse’s best response would be
A. “ I will get the physician to explain if your sex life will be affected by the surgery”
B. “Only your doctor can answer that, why don’t you ask him prior to discharge”
C. “ A transurethral prostatectomy does not usually result to erectile dysfunction”
D. “you would not be able to engage in sexual intercourse but you can express your love for your spouse by alternate acts such as cuddling”
Answer: C- prior to surgery, the client should be informed that his sexual functioning will not be hampered other than retrograde ejaculation, which is not physically harmful. NSNA NCLEX-RN REVIEW © 2000 page 842

Neuro 20 items:
CVA 5 items:
1. A post CVA client with right sided paralysis is admitted to the medical-intensive unit. Upon assessment, she is awake and alert but has difficulty speaking because of impairment of facial muscles involved with production of speech. When documenting the client’s speech, the nurse notes that the client has
A. Seimantic aphasia
B. Receptive aphasia
C. Dysarthria
D. Dysphagia
Answer: C- dysarthria is the term used to describe difficulty of speaking when muscle impairment is present. Seimantic aphasia is the inability to understand the meaning of the word. Receptive aphasia is the inability to understand spoken or written words. Dysphagia is the inability to swallow. NSNA NCLEX-RN REVIEW © 2000 page 748

2. Which nursing intervention has been found to be the most effective means of preventing plantar flexion in a client who has had a CVA with residual paralysis?
A. Place the client’s feet against a firm footboard
B. Reposition the client every 2 hours
C. Have the client wear ankle high tennis shoes at intervals all throughout the day
D. Massage the client’s feet and ankles regularly
Answer: C- the use of ankle high tennis shoes has been found to be the most effective in preventing plantar flexion (foot drop) because they add support to the foot and keep it in the correct anatomic position. Foot boards stimulate spasms and are not routinely recommended. Lippincott’s NCLEX-RN Review © 2005 page 416

3. An adult client with hemiplegia and right hemianopia expresses concern about how to operate the vacuum cleaner and washing machine at home. Which of the following nursing diagnoses is appropriate for this client?
A. High risk for injury related to right sided weakness
B. Impaired home maintenance management related to paralysis and visual impairment
C. Altered health management related to altered mobility and sensory perception
D. Hygiene and self-care deficit related to inability to keep the house clean
Answer: B- impaired home maintenance management is an appropriate diagnostic statement when a client with impaired vision or mobility expresses concern about ability to maintain the home properly. NSNA NCLEX-RN REVIEW © 2000 page 608

4. During recovery from CVA, a client is given nothing by mouth to help prevent aspiration. To determine if the client is ready for a liquid diet, the nurse assesses the client’s swallowing ability once each shift. This assessment evaluates
A. Cranial nerves I and II
B. Cranial nerves III and V
C. Cranial nerves VI and VIII
D. Cranial nerves IX and X
Answer: D- swallowing is a motor function of cranial nerves IX and X. cranial nerves I, II and VIII do not possess motor functions. The motor functions of cranial nerve III include extraocular eye movement, eyelid elevation and pupil constriction. The motor function of cranial nerve V is chewing and cranial nerve VI controls lateral eye movements. Springhouse NCLEX-RN Review Interactive

5. A client is experiencing mood swings after a CVA and often has episodes of tearfulness that are distressing to the family. Which is the best technique for the nurse to instruct family members to try when the client experiences a crying episode?
A. Sit quietly with the client until the episode is over
B. Ignore the behavior
C. Attempt to divert the client’s attention
D. Tell the client that this behavior is unacceptable
Answer: C- a client who has brain damage may be emotionally labile and may cry or laugh for no explainable reason. Crying episodes are best dealt with attempting to divert the client’s attention. Ignoring the behavior will not affect the mood swing or the crying and may increase the client’s sense of isolation. Telling the client to stop is inappropriate. Lippincott’s NCLEX-RN Review © 2005 page 417

Parkinson’s 5 items:
1. Which nursing diagnosis takes highest priority for a client with parkinson’s crisis?
A. Altered nutrition less than body requirements
B. Ineffective airway clearance
C. Altered urinary elimination
D. Risk for injury
Answer: B- in parkinson’s crisis, dopamine-related symptoms are severely exacerbated, virtually immobilizing the client. A client confined to bed during such crisis is at risk for aspiration and pneumonia. Also excessive drooling increases the risk of airway obstruction. Because of these concerns, the nursing diagnosis ineffective airway clearance takes the highest priority. Springhouse NCLEX-RN Review Interactive

2. The nurse is developing a teaching plan for a client newly diagnosed with parkinson’s disease. Which of the following topics that the nurse plans to discuss is the most important?
A. Maintaining balanced nutritional diet
B. Enhancing the immune system
C. Maintaining a safe environment
D. Engaging in diversional activities
Answer: C- the primary focus is maintaining a safe environment because the client with parkinson’s disease often has a propulsive gait, characterized by tendency to take increasingly quicker steps while walking. This type of gait often causes the client to fall or to have trouble stopping. Lippincott’s NCLEX-RN Review © 2005 page 417

3. A client with parkinson’s disease asks the nurse to explain to his nephew “what the doctor said the pallidotomy would do”. What is the nurse’s best response? The main goal for the client after pallidotomy is improved
A. Functional ability
B. Emotional stress
C. Alertness
D. Appetite
Answer: A- the goal of a pallidotomy is to improve functional ability for the client with parkinson’s disease. This is a priority. Pallidotomy creates lesions in the globus pallidus to control extrapyramidal disorders that affect control of movement and gait. Lippincott’s NCLEX-RN Review © 2005 page 417

4. A client with parkinson’s disease needs a long time to complete her morning hygiene, but she becomes annoyed when the nurse offers assistance and refuses all help. Which statement is the nurse’s best response in this situation?
A. Tell the client firmly that she needs assistance and help her with her care
B. Praise the client for her desire to be independent and give her extra time and encouragement
C. Tell the client that she is being unrealistic about her abilities and must accept the fact that she needs help
D. Suggest to the client that if she insists on self-care, she should modify her routine
Answer: B- ongoing self-care is a major focus for clients with parkinson’s. The client should be given extra time as needed and praised for her efforts to remain independent. Lippincott’s NCLEX-RN Review © 2005 page 417

5. In planning care for the client with advanced parkinson’s disease, which activity is most likely to be effective in alleviating fatigue?
A. Getting him to bed on time
B. Avoiding high carbohydrate foods
C. Collaborating with him when scheduling activities
D. Providing for morning and afternoon naps while he is in the hospital
Answer: C- scheduling activities in collaboration with the client with parkinson’s will allow him to proceed at his own pace and maximize his strength. NSNA NCLEX-RN REVIEW © 2000 page 670

Multiple sclerosis 5 items:
1. A client has had multiple sclerosis for 15 years and received various drug therapies. What is the primary reason why the nurse has found it difficult to evaluate the effectiveness of the drugs that the client has used? Clients with MS
A. Exhibit tolerance to many drugs
B. Experiences spontaneous remissions from time to time
C. Require multiple drugs simultaneously
D. Endure long periods of exacerbations before the illness responds to a particular drug
Answer: B- evaluating drug effectiveness is difficult because a high percentage of clients with MS exhibit unpredictable episodes of remission, exacerbation, and steady progress without apparent cause. Clients with MS do not necessarily have increased tolerance to drugs nor do they endure long periods of exacerbation before the illness responds to a particular drug. Lippincott’s NCLEX-RN Review © 2005 page 418

2. Which of the following is inappropriate for the nurse to include in the discharge plan for a client with MS who has an impaired peripheral sensation?
A. Carefully test the temperature of bath water
B. Avoid kitchen activities because of the risk of injury
C. Avoid hot water bottles and heating pads
D. Inspect the skin daily for injury or pressure points
Answer: B- the client should not be instructed to avoid kitchen activities out of fear of injury; independence and self-care are also important. However the client should meet with the occupational therapist to learn about assistive devices and techniques that can reduce injuries such as burns and cuts that are common in kitchen activities. Lippincott’s NCLEX-RN Review © 2005 page 418

3. Which of the following is an inappropriate outcome to establish with a client with MS?
A. The client will develop joint mobility
B. The client will develop muscle strength
C. The client will develop cognition
D. The client will develop mood elevation
Answer: C- care for the client with MS is directed towards maintaining joint mobility, preventing deformities, maintaining muscle strength and rehabilitation, preventing and treating depression and providing client motivation. Lippincott’s NCLEX-RN Review © 2005 page 419

4. The daughter of a client with multiple sclerosis asks the nurse what she can do at home to help her mother. Which of the following measures would be most beneficial?
A. Psychotherapy
B. Regular exercise
C. Weekly visits by another person with MS
D. Day care for the granddaughter
Answer: B- an individualized regular exercise program benefits the client to relieve muscle spasms. The client can be taught to use unaffected muscles to promote coordination because MS is a progressive debilitating condition. Lippincott’s NCLEX-RN Review © 2005 page 419

5. The nurse is preparing a client with MS for discharge from the hospital to home. Which of the following instructions is appropriate?
A. “ You will need to accept the necessity for a quiet and inactive lifestyle”
B. “ Keep active, use stress reduction strategies and avoid fatigue”
C. “ Follow good health habits to change the course of the disease”
D. “ Practice to use the mechanical aids that you will need when future disabilities arise”
Answer: B- the nurse’s most positive approach is to encourage the client with MS to keep active, use stress reduction strategies and avoid fatigue because it is important to support the immune system while remaining active. Lippincott’s NCLEX-RN Review © 2005 page 419

Head Injury 5 items:
1. The nurse is caring for a confused client who sustained a head injury resulting in subdural hematoma. The client’s blood pressure is 100/60 mm Hg and he is unresponsive. Select the most effective position for the client as the nurse transports him to the operating room
A. Semi-fowler’s position
B. Trendelenberg
C. High-fowler’s
D. Supine
Answer: A- the client’s head should be elevated at least 30 degrees to lower the increased intracranial pressure, which may be dangerously elevated in a subdural hematoma. The venous blood pressure begins to decline as the intracranial pressure rises. NSNA NCLEX-RN REVIEW © 2000 page 608

2. The nurse is performing a mental status examination on a client diagnosed with subdural hematoma. This test assesses which of the following?
A. Cerebral function
B. Intellectual function
C. Cerebellar function
D. Sensory function
Answer: A- the mental status function assesses functions governed by the cerebrum. Some of these are orientation, judgment, attention span and abstract reasoning. Springhouse NCLEX-RN Review Interactive

3. When caring for a client with head trauma, the nurse notes a small amount of clear, watery fluid oozing from the client’s nose. What should the nurse do?
A. Test the nasal drainage for glucose
B. Look for a halo sign after the drainage dries
C. Have the client blow the nose
D. Keep the client in supine position
Answer: A- because the CSF contains glucose, testing the nasal drainage for glucose helps determine whether the drainage is CSF. The nurse looks for halo sign only if the drainage is blood tinge. Springhouse NCLEX-RN Review Interactive

4. A client is at risk for increased intracranial pressure. Which of the following would be the priority of the nurse to monitor?
A. Unequal pupil size
B. Decreasing systolic BP
C. Tachycardia
D. Decreasing body temperature
Answer: A- increase ICP causes unequal pupils as a result of pressure on the third cranial nerve. Increase ICP causes an increase in the systolic pressure, which reflects the additional pressure needed to perfuse the brain. Lippincott’s NCLEX-RN Review © 2005 page 413

5. The client has signs of increased ICP. Which of the following is an early indicator of deterioration in the client’s condition?
A. Widening pulse pressure
B. Decrease pulse rate
C. Dilated fixed pressure
D. Decrease in the level of consciousness
Answer: D- a decrease in the client’s level of consciousness is an early indicator of deterioration of the client’s neurologic status. Changes in level of consciousness such as irritability and irritability may be subtle. Widening pulse pressure, decrease in pulse rate and dilated fixed pupils occur later if the increased ICP is not treated. Lippincott’s NCLEX-RN Review © 2005 page 413

Respi 20 items:
COPD 5 items:
1. An adult client who has chronic obstructive pulmonary disease needs frequent monitoring of arterial blood gases. Following the drawing of ABG, it is essential for the nurse to do which of the following?
A. Encourage the client to cough and deep breathe
B. Apply pressure to the puncture site for 5 minutes
C. Shake the vial of blood before transporting it to the laboratory
D. Keep the client on bed rest for 2 hours
Answer: B- it is essential to apply pressure to the puncture site for 5 minutes to ensure the client does not bleed from the arterial puncture. NSNA NCLEX-RN REVIEW © 2000 page 748

2. The nurse is interpreting the results of arterial blood gas analysis performed on an adult client. The values include pH 7.35, pCO2 60, HCO3 35, and O2 60. Which interpretation is most accurate?
A. The client is in metabolic acidosis
B. The client is in compensated metabolic alkalosis
C. The client is in respiratory alkalosis
D. The client is in compensated respiratory acidosis
Answer: D- a pH is of 7.35 is on the acid side of normal. All of the other values are abnormal so the client is compensated. The CO2 is sharply elevated and will lower the pH. The HCO3 is elevated and is responsible for bringing the pH up to the normal range. An abnormal O2 suggest that the problem is a respiratory one. NSNA NCLEX-RN REVIEW © 2000 page 748

3. A client with COPD is admitted to the M-S unit. To help this client maintain a patent airway and achieve maximal gas exchange the nurse should
A. Instruct the client to drink 2 liters of fluid daily
B. Maintain the client on bed rest
C. Administer anxiolytics as prescribed to control anxiety
D. Administer pain medications as prescribed
Answer: A- mobilizing secretion is crucial to maintaining a patent airway and maximizing gas exchange in a client with COPD. Measures that help mobilize secretions include drinking 2 liters of fluid daily, practicing purse-lip controlled breathing and engaging in moderate activity. Springhouse NCLEX-RN Review Interactive

4. When performing postural drainage, which of the following factors promotes the movement of secretions from the lower to the upper respiratory tract?
A. Friction between cilia
B. Force of gravity
C. Sweeping motion of cilia
D. Involuntary muscle contractions
Answer: B- the principle behind using postural drainage is that gravity will help move secretions from smaller to large airways. Postural drainage is best used after percussion has loosened secretions. Coughing or suctioning is then used to remove secretions. Lippincott’s NCLEX-RN Review © 2005 page 303

5. Which of the following blood gas abnormalities should the nurse anticipate in a client with advanced COPD?
A. Increased PaCO2
B. Increased PaO2
C. Increased pH
D. Increased oxygen saturation
Answer: A- as COPD progresses, the client typically develops increased PaCO2 levels and decreased PaO2 levels. This results in decreased pH and decreased oxygen saturation. These changes are the result of air trapping and hypoventilation. Lippincott’s NCLEX-RN Review © 2005 page 303

Chest tubes 5 items:
1. A client has a chest tube to a pleur-evac drainage system attached to wall suction. An order to ambulate the client has been received. To ambulate the client safely the nurse should
A. Clamp the chest tube and carefully ambulate the client in a short distance
B. Question the order to ambulate the client
C. Carefully ambulate the client, keeping the pleur-evac lower than the client’s chest
D. Disconnect the pleur-evac from the client’s chest tube , leave it attached to the bed, ambulate the client then reconnect the chest tube when the client returns to the bed
Answer: C- the pleur-evac must not be raised above the chest level because it can cause backflow of the fluid into the pleural space precipitating collapse of the lung or mediastinal shift. The pleur-evac must remain upright and the chest tube should not have traction on it. NSNA NCLEX-RN REVIEW © 2000 page 833

2. The nurse is assessing the client who has a chest tube to water seal drainage with a one bottle drainage system. Which of the following statements best indicate proper functioning of the chest drainage system?
A. The nurse observes bubbling from the straw in the bottle when the client coughs
B. The nurse observes no fluctuations in the straw in the bottle with respiration
C. The nurse observes rising and falling in the straw of the bottle with respiration
D. The nurse observes the air vent is clamped off
Answer: C- fluctuations of 5-10 cm during normal breathing are common. These fluctuations provide continuous manometer pressure changes in the pleural space and are a reliable indicator of overall respiratory effort. NSNA NCLEX-RN REVIEW © 2000 page 833

3. When caring for a client with a chest tube and water seal drainage system, the nurse should implement which of the following interventions?
A. Verify the air vent on the water seal drainage system is capped when the suction is off
B. Strip the chest drainage tubes at least every 4 hours if excessive bleeding occurs
C. Ensure that the chest tube is clamped when moving the client out of bed
D. Make sure that the drainage apparatus is always below the client’s chest level
Answer: D- the drainage apparatus is always kept below the client’s chest level to prevent back flow of fluid into the pleural space. The air vent must always be open in a closed chest drainage system to allow air from client to escape. Lippincott’s NCLEX-RN Review © 2005 page 306

4. A client who underwent lobectomy and has a water seal drainage system is breathing with a little more effort and at a faster rate than 1 hour ago. The client’s pulse rate is also increased. Which should the following actions should the nurse implement?
A. Check the tubing to ensure that the client is not lying or kinking it
B. Increase the suction
C. Lower the drainage bottles 2 to 3 feet below the level of the client’s chest
D. Ensure that the client’s chest tubes has 2 clamps on it to prevent air leaks
Answer: A- in this case there may be some obstructions to the flow of air and fluid out of the pleural space causing air and fluid to collect and build up pressure. This prevents the remaining lung from re-expanding and can cause mediastinal shift to the opposite side. The nurse’s first response is to assess the tubing for kinks or obstruction. Lippincott’s NCLEX-RN Review © 2005 page 306

5. A nurse observes a constant gentle bubbling in the water seal column of a water seal chest drainage system. This observation prompts the nurse to do which of the following?
A. Continue monitoring as usual, this is expected
B. Check the connectors between the chest and drainage tubes and where the drainage tube enters the collection bottle
C. Decrease the suction to 15 cm H2O and continue observing the system for changes in bubbling during the next several hours
D. Drain half of the water from the water seal chamber
Answer: B- there should never be constant bubbling in the water seal bottle; normally the bubbling is intermittent. Constant bubbling in the water seal indicates air leak, which means that less negative pressure is being exerted on the pleural space. Lippincott’s NCLEX-RN Review © 2005 page 306

Asthma 5 items:
1. A client recovering from acute asthma attack experiences respiratory alkalosis. The nurse measures the respiratory rate of 46 breaths /minute, heart rate of 110 bpm, BP of 160/90 mm Hg, and a temperature of 98.6 F (37 C). to help correct respiratory alkalosis, the nurse should
A. Insert a nasogastric tube as ordered
B. Administer acetaminophen (Tylenol) as prescribed
C. Administer antibiotics as prescribed
D. Instruct the client to breathe into a paper bag
Answer: D- a client recovering from acute asthma attack who experiences respiratory alkalosis should be instructed to breathe into a paper bag to increase arterial carbon dioxide tension and ease anxiety (which may exacerbate the alkalosis). Springhouse NCLEX-RN Review Interactive

2. A 34 year old client with a history of asthma is admitted to the emergency department. The nurse notes that the client is dyspneic, with a respiratory rate of 35 breathes/minute, nasal flaring and use of accessory muscles. Auscultation of the lung fields reveals greatly diminished breath sounds. Based on the findings what action should the nurse to initiate care of the client?
A. Initiate oxygen therapy and reassess the client in 10 minutes
B. Draw blood for ABG analysis and send the client for chest x-ray
C. Encourage the client to relax and breathe slowly through the mouth
D. Administer bronchodilators
Answer: D- in an acute asthma attacks, diminished breath sounds can be ominous sign indicating lack of air movement and impending respiratory failure. The client requires immediate interventions with inhaled bronchodilators, intravenous corticosteroids and possibly intravenous theophylline. Lippincott’s NCLEX-RN Review © 2005 page 303

3. The nurse would anticipate which of the following ABG results in aclient experiencing a prolonged, severe asthma attack?
A. Decreased PaCO2, increased PaO2, decreased pH
B. Increased PaCO2, decreased PaO2, decreased pH
C. Increased PaCO2, increased PaO2, increased pH
D. Decreased PaCO2, decreased PaO2, decrease pH
Answer: B- as severe asthma attack worsens, the client becomes fatigued and alveolar hypoventilation develops. This leads to carbon dioxide retention and hypoxemia. The client develops respiratory acidosis. Lippincott’s NCLEX-RN Review © 2005 page 304

4. Which of the following health promotion activities should the nurse include in the discharge teaching plan for a client with asthma?
A. Incorporate physical exercise as tolerated into the daily routine
B. Monitor peak flow numbers after meals and at bed time
C. Eliminate stressors in the work and home environment
D. Use sedatives to ensure uninterrupted sleep at night
Answer: A- physical exercise is beneficial and should be incorporated as tolerated in to the client’s schedule. Lippincott’s NCLEX-RN Review © 2005 page 304

5. Which of the following findings would most likely indicate the presence of a respiratory infection in a client with asthma?
A. Cough productive of yellow sputum
B. Bilateral expiratory wheezing
C. Chest tightness
D. Respiratory rate of 30 breaths / minute
Answer: A- cough productive of yellow sputum is the most likely indicator of a respiratory infection. The other signs and symptoms –wheezing, chest tightness, and increased respiratory rate are all findings associated with asthma attack. Lippincott’s NCLEX-RN Review © 2005 page 304
Psychiatric Exam

1. A client who is acutely ill and is hospitalized with metastatic lung carcinoma begs for a pass to attend a son’s high school graduation in a city 100 miles from the hospital. “if only I could do this one thing, then I’ll be ready to let go,” the client says. The nurse identifies this behaviour as an example of:
a. Being unrealistic and denying the degree of illness.
b. Using bargaining as a reaction to death and dying.
c. Being manipulative to get own way.
d. Being unaware of the diagnosis.
RATIONALE: (2) refer to Elisabeth Kubler-Ross’s emotional stages of death and dying. The client is being neither unrealistic (Answer 1), manipulative (Answer 3), nor unaware of the diagnosis (Answer 4).
Reference: DAVIS’S NLCEX RN Success Second Edition, page 464
2. A nursing care plan for a hospitalized client who is hyperactive in a manic episode must include:
a. Involvement in a group activity and encouragement to talk.
b. Permissive acceptance of bizarre behaviour.
c. Protection against suicide.
d. Attention to adequate food and fluid intake.
RATIONALE: (4) during the manic episode, the client may be too busy to eat and sleep. Answer 1 and 3 are more appropriate for schizophrenia with bizarre behaviour. Clients in manic episode exhibit hyperactive behaviour.
Reference: DAVIS’S NLCEX RN Success Second Edition, page 464
3. What is the main consideration in planning the general care of clients with dementia is that:
a. Their capacity for physical activity is diminished.
b. They be protected from suicide attempts.
c. Team effort be aimed at increasing their independence.
d. The staff be sympathetic when clients mention their failing abilities.
RATIONALE: (1) An important principle to remember here is that a program of care should not increase physiological losses by overtaxing the client’s physical capacities. It is important to remember that suicide is the main concern in depression and not a key concern in dementia, as in Answer 2. Empathy rather than sympathy (Answer 4) is a helpful behaviour. Answer 3, increasing independence, is not realistic; maintaining it is.
Reference: DAVIS’S NLCEX RN Success Second Edition, page 465
4. A crisis nurse explains to a colleague that the main concern of treatment in crisis intervention is on the:
a. Present and on restoration to the usual level of functioning.
b. Past and on freeing the unconscious.
c. Past in relation to the present.
d. Present and on the repression of unconscious drives.
RATIONALE: (1) To resolve immediate problems, focus on a person’s ability to cope and usual level of functioning. Answer 2 is incorrect because it is a psychoanalytic focus. Answer 3 and 4 are incorrect because they are the focus of brief psychotherapy.
Reference: DAVIS’S NLCEX RN Success Second Edition, page 466
5. What are the most significant signs and symptoms indicative of alcohol withdrawal delirium?
a. Decreased BP and pulse, restlessness.
b. Increased BP and pulse, seizures.
c. Cramps, nausea, vomiting.
d. Anorexia, diarrhrea, dehydration.
RATIONALE: (2) Alcohol abuse is the problem here. Answer 1 is incorrect because although the client with delirium tremens is often restless, vital signs are increased (BP and pulse) and the client may go into seizures if not treated medically as soon as possible. Answer 3 is incorrect because it focuses on GI signs and symptoms; think neurological and cardiovascular systems for the most significant symptoms in this case. Answer 4 is incorrect because it focuses on GI signs and symptoms rather than significant neurological and cardiovascular systems.
Reference: DAVIS’S NLCEX RN Success Second Edition, page 466
6. A client is diagnosed as having a paranoid disorder. What implication might this have for a nurse?
a. Let the client talk about the suspicions without correcting misinformation.
b. Argue with the client about his or her ideas.
c. Placate the client by agreeing with what he or she says.
d. Avoid talking to other nurses when the client can see them but cannot hear what is being said.
RATIONALE: (4) A client with paranoid disorder is suspicious, so a nurse must make every effort not to engage in behaviour the client can misinterpret. Answer 1 and 3 are incorrect because a nurse should give correct information about what the client says and not placate him or her (the client will sense the falseness). Answer 2 is incorrect because arguing just solidifies the client’s ideas. In a neutral voice, the nurse should give correct information.
Reference: DAVIS’S NLCEX RN Success Second Edition, page 467
7. Which is the common characteristic behaviour of a panic response is likely to be noted?
a. Goal-directed behaviour aimed at a “flight” from apparent threat.
b. Automatic behaviour with poor judgment.
c. A severity of reaction that is not related to the severity of the threat to self-esteem.
d. A delayed reaction in perceiving the danger.
RATIONALE: (2) In panic, a person is highly suggestible and follows “herd instinct” rather than exercising independent judgment and goal directed problem solving (Answer 1). Answer 3 is incorrect because the severity of the reaction is related to the severity to the threat. The more severe the perceived threat 9actual or imagery), the more intense the reaction to the danger, and not delayed as in Answer 4.
Reference: DAVIS’S NLCEX RN Success Second Edition, page 468
8. What are the significant side effects when a client is taking Haloperidol (Haldol)?SATA
a. Diarrhea
b. Constipation
c. Orthostatic hypotension
d. Urinary retention
e. Decreased appetite
f. Elevated blood pressure
RATIONALE: (2, 3 and 4) Answer 2 is correct because Haldol does produce an anticholinergic response, such as dry mouth and constipation, as well as orthostatic hypotension and urinary retention. Answer 3 and 4 are correct because these are side effects are seen when on Haldol. Answer 1 is incorrect because Haldol does not typically cause diarrhea. Answer 5 is incorrect because clients taking Haldol experience an increase in appetite, not a decrease. Answer 6 is incorrect because high BP is not commonly seen in clients taking Haldol.
Reference: DAVIS’S NLCEX RN Success Second Edition, page 468
9. Which observation of a nurse in a psychomotor manifestation of anxiety will be included?
a. Decreased activity.
b. Increased activity.
c. Increased lability of emotions.
d. Decreased lability of emotions.
RATIONALE: (2) Research shows that increased activity is a psychomotor manifestation of anxiety; therefore, Answer 1 is incorrect. Answer 3 and 4 are incorrect because emotions is not a psychomotor manifestation.
Reference: DAVIS’S NLCEX RN Success Second Edition, page 468
10. The nurse needs to know that anxiety may increase intellectual functioning because anxiety may:
a. Increase the perceptual field.
b. Increase ability to concentrate.
c. Decrease the perceptual field.
d. Decrease random activity.
RATIONALE: (3) Research shows that anxiety decrease the perceptual field; therefore, Answer1 is incorrect. Research also shows that anxiety decreases the ability to concentrate and increases random activity; therefore, Answer 1, 2 and 4 are incorrect.
Reference: DAVIS’S NLCEX RN Success Second Edition, page 468
11. Based on Ericskon what is the growth and development primarily of an old age?
a. Ego integrity versus despair.
b. Autonomy versus shame and doubt.
c. Trust versus mistrust.
d. Industry versus inferiority.
RATIONALE: (1) This is correct according to Erickson. Answer 2 relates to toddler; Answer 3, to infancy; Answer 4, to latency period in childhood.
Reference: DAVIS’S NLCEX RN Success Second Edition, page 469
12. A high risk client of suicide needs to be assess with which suggested behaviour?
a. Major depression with melancholia features.
b. Schizophrenic disorders.
c. Bipolar mood disorders, manic episode.
d. Psychological factors affecting physical condition.
RATIONALE: (1) Suicide risk is highest in clients with major depression. Answer 2, 3 and 4 are incorrect because these diagnoses alone do not indicate suicide risk as does the diagnosis of depression.
Reference: DAVIS’S NLCEX RN Success Second Edition, page 469
13. What is the overall goal of nursing intervention when working with a person who is anxious?
a. Remove anxiety.
b. Develop the person’s awareness of anxiety.
c. Protect the person from anxiety.
d. Develop the person’s capacity to tolerate mild anxiety and to use it constructively.
RATIONALE: (4) Some anxiety is necessary to learn. Answer 2 is incorrect because it is incomplete. Answer 1 and 3 are incorrect because anxiety is necessary for learning and growth.
Reference: DAVIS’S NLCEX RN Success Second Edition, page 469
14. As a nurse which is the suggested activity to a hyperactive behaviour client?
a. Solitary activity, such as reading.
b. Hammering on metal in a jewelry-making class.
c. Playing chess.
d. Competitive games.
RATIONALE: (2) It will provide energy release without the external stimuli and pressure of competitive games (Answer 3 and 4). Reading (Answer 1) usually reuires sitting, which a client who is hyperactive cannot readily do.
Reference: DAVIS’S NLCEX RN Success Second Edition, page 469
15. A person is assessed for signs of apraxia when there is impaired ability to:
a. Recognized or understand familiar words, objects, or people despite intact sensory function.
b. Express thoughts in writing due to organic cerebral pathology.
c. Speak, either a loss or deterioration of language, or incorrect use of words, with excessive use of indefinite terms. (e.g., “thing,” “it”).
d. Carry out purposeful motor activities despite intact motor abilities, sensory function, and comprehension of the required task (e.g., inability to use utensils).
RATIONALE: (4) This is the correct definition. Answer 1 is incorrect because it defines agnosis. Answer 2 is incorrect because it defines agraphia. Answer 3 is incorrect because it defines aphasia.
Reference: DAVIS’S NLCEX RN Success Second Edition, page 470
16. A client who is elderly has dementia related to cerebral arteriosclerosis says to the nurse, “I’m going to the university today to be their guest lecturer on aerodynamics.” Which response by the nurse would be most therapeutic?
a. “Do you know that you are in the hospital now?”
b. “Are you saying that you would like to be asked to give a lecture at the university?”
c. “How about watching a movie on television instead?”
d. “It’s more important that you don’t tire yourself out.”
RATIONALE: (2) The best of four poor choices here is an attempt to understand what the client means. The other choices are not helpful. Answer 3 and 4 switch the focus and ignore the client’s statement; Answer 1 is too brusque an attempt to bring the client back to reality.
Reference: DAVIS’S NLCEX RN Success Second Edition, page 463
17. A client begins having auditory hallucinations. When the nurse approaches, the client whispers, “Did you hear that terrible man? He is scary!” which would be the best response for the nurse to make initially?
a. “Who is he? Do you know him?”
b. “I don’t hear a man’s voice, but you look scared.”
c. “What is he saying?”
d. “I don’t hear anything. What scary things is he saying?”
RATIONALE: (2) This is reality-based response, as well as one that acknowledge the client’s nonverbal reaction. The other choices are not the best because Answer 1 and 3 focus on “voice,” which reinforces the hallucination, and no doubt is raised; in Answer 2 doubt is raised, but he focus is on “voice” and not on client’s feelings.
Reference: DAVIS’S NLCEX RN Success Second Edition, page 463
18. After seeing a number of doctors for nonspecific complaints of chest pains, with no conclusive findings of organic disease, a client is referred to a local mental health center. The client has read extensively about coronary disease and talks continuously about the symptoms in great detail. Which approach by the nurse would be best when meeting this client for the first time?
a. Comment on a neutral topic instead of using the usual conservation opener of “How are you today?”
b. Allow the client to describe the physical problems to become familiar with them.
c. Give the client a simple but direct explanation of the physiological basis for the symptoms.
d. Let the client know that the nurse is familiar with the psychogenic problems and guide the discussion to other areas.
RATIONALE: (2) It shows acceptance by listening to the client’s initial account of the physical problems. Neither a superficial focus nor a technical explanation of physical problems (Answer 1 and 3) conveys acceptance of the client as a person; Answer 4 is too abrupt for an initial response.
Reference: DAVIS’S NLCEX RN Success Second Edition, page 463
19. A client who is agitated begins to shout insults and threats at others, and start demolishing the recreation room. What is the best response or action by the nurse?
a. Firmly set limits on the behaviour.
b. Allow the client to continue, because the client is seeking to express herself or himself.
c. Tell the client he or she is trying to intimidate other clients.
d. Let the client know that he or she does not need to express anger at the nurse by demolishing the recreation room.
RATIONALE: (1) the nurse needs to set limits to ensure the safety of the client and others. Answer 2 is incorrect because the client may hurt herself or himself or others. Answer3 and 4 are incorrect because they are interpretations that may or may not be correct, and they do not control the unsafe behaviour.
Reference: DAVIS’S NLCEX RN Success Second Edition, page 464
20. What feeling tone is the nurse most likely to see the client demonstrate during major depression with psychotic features?
a. Suspicion
b. Agitation
c. Loneliness
d. Worthlessness
RATIONALE: (4) Feelings of worthlessness or low self-esteem are the underlying problem in depression. Answer 1, 2 and 3 may occur but are not most likely to be seen.
Reference: DAVIS’S NLCEX RN Success Second Edition, page 466
21. While the nurse is interviewing a teenage client, the client says, “I suppose you have to tell my parents everything.” What would be the best response by the nurse?
a. “What are you going to tell me that is so secret that I can’t tell your parents?”
b. “If you tell me you are going to do something to hurt yourself I will have to tell your parents, but I will tell you first before telling them.”
c. “Everything you tell me I will is confidential. I will not tell your parents anything”
d. “Everything you tell me I will need to tell your parents. They have the right to know.”
RATIONALE: (2) Confidentiality cannot be guaranteed if there is a danger to the client r to others. Answer 1 is incorrect because it is a sarcastic response and denies that the client has serious problems. Answer 3 contradicts Answer 2. Answer 4 is incorrect because parents do not have the right to know the content of therapy sessions except in the circumstances described in Answer 2.
Reference: DAVIS’S NLCEX RN Success Second Edition, page 464
22. A man is hospitalized following a car accident in which his wife died, and he is unable to attend the funeral due to his severe chest pain injuries. What would the nurse consider in forecasting this surviving spouse’s potential for difficulty with grief resolution?
a. Feeling of anger toward the hospital staff for keeping him hospitalized during the funeral.
b. Feelings of anger toward himself for having been injured but not killed in the accident.
c. His inability to participate in the cultural rituals of grief, wherein the reality of his wife’s death is emphasized.
d. His preoccupation with his own physical distress at this time.
RATIONALE: (3) One way to enhance the development of unresolved grief is not to participate in activities that demonstrate death. Answer 1, 2, and 4 may occur, but they are not the best predictors of grief resolution difficulties.
Reference: DAVIS’S NLCEX RN Success Second Edition, page 465
23. The nurse discovers a client crouched in a corner, looking pale and frightened, and holding a gushing wrist wound. A razor is nearby on the floor. What should the nurse do first?
a. Sit down on the floor, next to the client, and in a quiet, reassuring tone, say, “You seem frightened. Can I help you?”
b. Ask the aide to watch the client and run to get the doctor.
c. Apply pressure on the wrist, saying to the client, “You are hurt. I will help you.”
d. Go back down the hall to get the emergency cart.
RATIONALE: (3) Take care of the bleeding (physical aspects of care) first. In Answer 1 and 2, the client could suffer extensive blood loss if the nurse focuses on feelings at this point or leaves the client without first attempting to control the bleeding. In Answer 4, the client is left alone, bleeding, frightened, and with a razor still at the side.
Reference: DAVIS’S NLCEX RN Success Second Edition, page 465
24. An important part of the nursing care for a client with dementia would be:
a. Minimizing regression.
b. Correcting memory loss.
c. Rehabilitating toward independent functioning.
d. Preventing further deterioration.
RATIONALE: (1) Use of regression as a defense response can be minimized. Memory loss is usually permanent, not correctable; thus Answer 2 is wrong. However, disorientation attributed to loss of memory can be minimized. Clients usually become more dependent in the course of illness and deteriorate progressively. Thus Answer 3 and 4 are incorrect.
Reference: DAVIS’S NLCEX RN Success Second Edition, page 465
25. When a client has dementia, it is most important that the nurse plan the daily activities to:
a. Be highly structured.
b. Be changed each day to meet the client’s needs for variety.
c. Be simplified as much as possible to avoid problems with decision making.
d. Provide many opportunities for making choices to stimulate the client’s involvement and interest.
RATIONALE: (1) Clients with dementia feel more secure when they can count on their environment being the same, predictable, and consistent in detail from day to day (hence, structured) to compensate for feelings of loss of the familiar in terms of body functions, social environment, and so on. Answer 2 and 4 imply change not routine. Answer 3 although correct, is not the most important.
Reference: DAVIS’S NLCEX RN Success Second Edition, page 466
26. Which feeling is the nurse likely to identify as antecedent of self-destructive behaviour?
a. Omnipotence
b. Grandiosity
c. Low self-esteem
d. Self-satisfaction
RATIONALE: (3) When feelings of low self-esteem are prevalent, self-destructive behaviour reaches its peak. Hence, Answer 1, 2 and 4 are incorrect.
Reference: DAVIS’S NLCEX RN Success Second Edition, page 466
27. A client has a somatoform disorder, paralysis of the arm. It would not be helpful for the nurse to use logic and reason to divert this client’s attention from this physical state and because:
a. The client is not in contact with reality and thus is unable to “hear” or understand the nurse.
b. The client may need the symptoms to handle feelings of guilt or aggression.
c. The nature of the client’s particular illness makes the client suspicious of all medical personnel.
d. Paralysis of the arm has become a habitual response to stress.
RATIONALE: (2) This disorder is an attempt to cope with stress and is better choice than Answer 4, which may be true but is a tangential and irrelevant reason. Answer 1 is not correct because the client is aware of reality but may not understand the cause for the somatoform disorder. Answer 3is more relevant in a paranoid reaction.
Reference: DAVIS’S NLCEX RN Success Second Edition, page 466
28. To relate therapeutically with a client who is dependent on alcohol, it is important that the nurse base care on the understanding that alcohol dependence:
a. Is hereditary
b. Is due to lack of willpower and true remorse.
c. Results in always breaking promises.
d. Cannot be cured
RATIONALE: (4) Arrest of the disease is possible through abstinence, not through change in psychological response to alcohol. Answer 1, 2 and 3 are stereotyped statements, not generally or universally accepted.
Reference: DAVIS’S NLCEX RN Success Second Edition, page 466
29. The nurse needs to assess a client for depression. What are the most characteristic signs and symptoms of depression? SATA
a. Diarrhea
b. Constipation
c. Sleep disturbance
d. Poor appetite
e. Increased appetite
f. Anhedonia
RATIONALE: (2,3,4,6) These are symptoms of depression. Answer 1 is incorrect because this symptom is not commonly seen in clients who are depressed. Answer 5 is incorrect because this symptom is not commonly seen in clients who are depressed.
Reference: DAVIS’S NLCEX RN Success Second Edition, page 464
30. Which nursing intervention is effective when clients are severely anxious?
a. Encourage group participation.
b. Give detailed instructions before treatment procedures.
c. Impart information succinctly and concretely.
d. Increase opportunities for decision making.
RATIONALE: (3) Brief and specific information can be processed during severe anxiety. In severe anxiety, the person cannot respond to the social environment (as in Answer 1); giving detailed information results in overload, because the client cannot retain and recall data (as in Answer 2). Only directive information that is brief and specific is effective when the client cannot focus on what is happening. Decision making needs to be postponed until the person is less anxious; hence Answer 4 is wrong.
Reference: DAVIS’S NLCEX RN Success Second Edition, page 468
31. A new nurse is assigned to take clients for an outing. A client with an antisocial disorder approaches the nurse and says, “I like you. I’m glad you’ll be the one to take us out. My doctor told me that I can go too.” Which initial response by the nurse is best?
a. “Since I am new here and not familiar with unit routine, I will go check with the staff and be back.”
b. “It’s a beautiful day, and I’m glad that you have ground privileges now.”
c. “When did the doctor tell you that?”
d. “You seem pleased.”
RATIONALE: (1) This response aims to prevent use of manipulative patterns. In Answer 2, 3 and 4, the nurse needs to seek validation, and these response indicate acceptance without validation.
Reference: DAVIS’S NLCEX RN Success Second Edition, page 468
32. A client whose significant other recently died shows signs of grief resolution when he or she:
a. Wants to enter into another relationship soon.
b. Talks of both the positive and negative aspects of their relationship.
c. Makes up for deficiencies in the relationship, saying, “Things would have been better if we had only had more time.”
d. Expresses anger towards the deceased.
RATIONALE: (3) When the mourner can pass through the idealization stage and be more realistic about the positive and negative aspects of the loss, resolution of grief is beginning. Answer 3 and 4 occur earlier stages of grief. Answer 1 could be a sign of denial of grieving, an initial grief reaction.
Reference: DAVIS’S NLCEX RN Success Second Edition, page 468
33. A client says to his mother, “You are controlling me.” The mother asks the nurse what he may have meant. What is the best response by the nurse?
a. “He is upset and thinks you are taking charge of him.”
b. “He resents always having to meet your expectations.”
c. “I can’t tell you. You will have to ask him.”
d. “I think you can ask your son that. Do you want me to stay with you while you ask him?”
RATIONALE: (4) The mother needs to ask the client, not the nurse, but the nurse should also support the mother and encourage her to interact with her son. Answer 1 and 2 are incorrect because they are interpretations that state opinions about the client without his participation. Answer 3 is incorrect because no support is given to the mother.
Reference: DAVIS’S NLCEX RN Success Second Edition, page 469
34. A client who attempted suicide recently remarked to the nurse the next morning, “Let’s not think about that now. Maybe I’ll feel like thinking about it later.” The nurse identifies this as:
a. Blocking
b. Denial
c. Suppression
d. Repression
RATIONALE: (3) By definition, this is the conscious, deliberate effort to avoid talking or thinking about painful, anxiety-producing experiences. Answer 1, 2 and 4 are incorrect terms for the example given in the stem.
Reference: DAVIS’S NLCEX RN Success Second Edition, page 469
35. In conducting an assessment interview, the nurse needs to be aware that self-destructive behaviour is determined by:
a. A variety of factors, with the same factors present in each individual.
b. Genetic disturbances.
c. Interpersonal disturbances.
d. A variety of factors, different for each individual.
RATIONALE: (4) A variety of factors can cause self-destructive behaviour, and these differ for each individual. Hence, Answer 2 and 3 are not the best choice because, although correct, they are examples of a variety of factors; Answer 1 is the opposite of the correct answer.
Reference: DAVIS’S NLCEX RN Success Second Edition, page 469
36. The nurse observes for signs of heroin withdrawal, which may include:
a. Rhinorrhea, sneezing, and high fever.
b. Pupillary dilation, diaphoresis, and weight loss.
c. Papillary constriction, vomiting, and puritus.
d. Choreiform movements and frequent lip wetting.
RATIONALE: (2) Note the eyes; when a person is on heroin, the pupils are constricted; during withdrawal, they are dilated. Withdrawal does not usually include high fever, pupillary constriction, or choreiform movements, as in Answer 1, 3 and 4.
Reference: DAVIS’S NLCEX RN Success Second Edition, page 470
37. The nurse will look for what likely outcome of methadone treatment for heroine abuse and dependence?
a. Sedation
b. Euphoria
c. Neuritis
d. Blocking of the euphoric effect of heroine and elimination of craving
RATIONALE: (4) Methadone is a synthetic narcotic and has no euphoric effect. Methadone does not produce sedation (Answer 1), euphoria (Answer 2), or neuritis (Answer 3).
Reference: DAVIS’S NLCEX RN Success Second Edition, page 470
38. A client who is acutely agitated becomes increasingly aggressive. The staff’s verbal attempts to stop the aggressive behaviour are not effective. The client begins to shout threats at the staff and other clients, throws furniture’s, breaks windows, and hits, kicks, and bites other clients and staff. The client has a prn order for medication when agitated. Which action should the nurse take initially?
a. Orient the client to reality and place the client in a well-lit, quiet room.
b. Give the ordered tranquilizer and put the client in bed with the side rails up.
c. Lock the client in his or her room and call the doctor.
d. Have at least two staff members physically restrain the client and take the client to a quiet room.
RATIONALE: (4) With concern for danger to the other clients, staff and the environment, it is essential for the client to be restrained at this time. The other choices are incorrect because orientation and a quiet environment alone do not provide for safety when the client’s agitation is out of control (Answer 1); the initial delay in onset of effectiveness of the tranquilizer does not immediately provide for the safety needs of other clients, staff and environment (Answer 2); and locking the client in his or her room eliminates only the danger to others-additional measures would be needed to provide for the safety of this client (Answer 3).
Reference: DAVIS’S NLCEX RN Success Second Edition, page 470
39. Several staff members voice their frustrations about a client’s constant questions, such as “Should I go to the dayroom or should I stay in my room?” and “Should I have a cup of tea or a cup of coffee?” Which interpretation about this behaviour will help the nursing staff deal effectively?
a. The client’s inability to make decisions reflects a basic anxiety about making a mistake and being a failure.
b. The client’s indecisiveness is aimed at testing the staff’s reaction and acceptance of him or her.
c. The client’s dependence on others (staff) is a symptom that needs to be interrupted by firm limiting setting.
d. The client’s need to ask questions is a bid for attention.
RATIONALE: (3) Limit setting is an important intervention with a client who exhibits excessive, constant dependence on others for simple, seemingly inconsequential decisions in everyday life. The other choices are wrong because the situation presented provides insufficient data on which to base interpretations related to fear of failure (Answer 1), a need to test for staff acceptance (Answer 2), or bid for attention (Answer 4).
Reference: DAVIS’S NLCEX RN Success Second Edition, page 470
40. The nurse can anticipate that the person most likely to be at risk for depression is:
a. A person who is elderly with previous depressive episodes.
b. A man who is middle-aged and who is moderate alcohol drinker.
c. A housewife with three school-aged children.
d. A nursing student at exam time.
RATIONALE: (1) Is correct because depressive episodes are often recurrent. Answer 2, 3 and 4 may or may not experience depression. Answer 2 and 4 are more likely to experience anxiety. There does not appear to be any reason to expect that a housewife with three school children in school would be at risk for any particular emotional distress (Answer 3).
Reference: DAVIS’S NLCEX RN Success Second Edition, page 470
41. In admitting a client with Alzhiemer’s to the unit, which placement variable would have the highest priority?
a. Place the client with a roommate.
b. Place the client without roommate.
c. Place the client close to the nurses’ station.
d. Place the client a distance from the nurses’ station.
RATIONALE: (3) Nursing observation is easier if the client with Alzheimer’s is nearby. Therefore, Answer 4 is incorrect. A roommate may (Answer 1) or may not (Answer 2) be alright, but facilitating nursing observation is the highest priority for a client with memory problems and confusion who is at risk for wandering off the unit.
Reference: DAVIS’S NLCEX RN Success Second Edition, page 470
42. Which nursing action would be best for a client who is hospitalized, and is constantly upset with the staff, easily angers, and frequently shouts at the nurses?
a. Request that the client be moved to another unit.
b. Schedule a conference with the MD, nurse manager, and client about this behaviour.
c. Contact social services to meet with the client and family about the problem.
d. Involve the client and family in the development of the care plan.
RATIONALE: (4) The client is probably experiencing a loss of control over the hospitalization situation. Initially the client and family should be included in planning care, which returns decision making to the client and decreases frustration. Answer 1 and 3 shift the problem rather than attempt to resolve the issue. Answer 2 involves he client but brings in an authority figures (the MD), which may be seen as punishment or “parental” by the client
Reference: DAVIS’S NLCEX RN Success Second Edition, page 471

43. A client hospitalized with Alzhiemer’s disease is often found wandering in the streets. What measure(s) should be taken in the unit to prevent the client from wandering off?
a. Place the client in daytime restraints.
b. Place the client in night time restraints.
c. Provide a security guard at the door.
d. Use electronic surveillance devices.
RATIONALE: (4) This answer is concerned with accident prevention and is a means of observation of the client. Answer 1 and 2 are incorrect because the use of restraints is inappropriate and not justified. Having a security guard is not realistic (Answer 3).
Reference: DAVIS’S NLCEX RN Success Second Edition, page 471
44. During a home health assessment visit to an 84-year-old client living alone, which aspect of lifestyle noted by the nurse would be of greatest concern?
a. The family visits only twice a month.
b. The family maintains only phone contacts daily.
c. The client uses a cordless telephone rather than a standard phone.
d. The client prefers not to attend a senior citizen center for meals and recreation.
RATIONALE: (1) This is the correct answer because it lets the nurse know that contacts with the person who is elderly are infrequent and that ongoing assessment of needs and hoelth concerns is not likely to occur. Answer 2 is incorrect because it is not the greatest concern, as long as there is the opportunity for assessment by phone. A cordless phone is a good idea rather than a great concern (Answer 3). Although isolation is further increased by not wanting to go to a senior citizen center, it is not of greatest concern (Answer 4).
Reference: DAVIS’S NLCEX RN Success Second Edition, page 470
45. Since the death of her infant, a woman has lost weight, will not eat, spends most of her time immobile, and speaks only monosyllabic responses. She pays little attention to her appearance. One afternoon, this client comes to lunch with her hair combed and traces of lipstick. What could the nurse say to reinforce this change of behaviour?
a. “What happened? You combed your hair!”
b. “This is the first time I’ve seen you look so good.”
c. “You must be feeling better. You look much better.”
d. “I see that your hair is combed and you have lipstick on.”
RATIONALE: (4) a simple acknowledgement of what the nurse sees is the best response. Answer 3 makes an assumption that the client feels better if she looks better. Answer 1 and 2 can be taken as a put-down.
Reference: DAVIS’S NLCEX RN Success Second Edition, page 464
46. A 52-year-old who appears lucid learns that after surgery, he will wake up in the recovery room without his thick glasses and hearing aid. He immediately states that without these he will be confused and upset. The nurse determines that the client is trying to say that he:
a. Has periods of confusion and may have a psychiatric problem.
b. Is psychologically dependent on the hearing aid and glasses.
c. Needs the hearing aid and glasses to correctly perceive what is going on around him, and misperception will cause confusion.
d. Needs the hearing aid and glasses because he wants to be sure people are taking proper care of him.
RATIONALE: (3) A person with limited hearing and sight becomes confused and disturbed. Answer 1 and 4 are unwarranted assumptions. Answer 2 is incorrect because the client’s situation is not psychological dependency but actual sensory need.
Reference: DAVIS’S NLCEX RN Success Second Edition, page 466
47. A college student was doing exceptionally well and was complicated by her chemistry professor, who arranged for her to become his lab assistant and to do advanced research. Although very thorough in her work, when given constructive criticism this student becomes angry and stalks out the lab for a few hours. The most plausible theoretical explanation is that the student:
a. Knows she is right.
b. Thinks the professor is jealous of her.
c. Needs to feel and know that she is perfect.
d. Feels anxiety as a result of a threat to her security and self-image.
RATIONALE: (4) This is the all-inclusive answer. Answer 1, 2 and 3 may be part of Answer 4
Reference: DAVIS’S NLCEX RN Success Second Edition, page 465
48. A teenager with acting-out behaviours says to the nurse, “I want you to go tell the teacher I am sick and I am to be allowed to do what I want.” The nurse determines that this statement best represents:
a. Insight
b. Manipulation
c. Dependency
d. Trust
RATIONALE: (2) Manipulation is the attempt to control the behaviour of others to achieve one’s own goals. Answer 1, insight, is understanding and using understanding to correct one’s behaviour. Answer 3 and 4 are incorrect because although the client may be trying to con the nurse into believing he or she needs the nurse to do something for him or her and the client trusts the nurse, the real purpose of the request is manipulation.
Reference: DAVIS’S NLCEX RN Success Second Edition, page 466
49. A client is in withdrawn catatonic state and exhibits waxy flexibility. During the initial phrase of hospitalized for this client, the nurse’s first priority is to:
a. Identify the predisposing factors in the illness.
b. Encourage the client to discuss concerns that led to the catatonic state.
c. Provide a warm, nurturing relationship, with therapeutic use of touch.
d. Watch for edema and cyanosis of the extremities.
RATIONALE: (4) Circulation may be severely impaired in a client with waxy flexibility who tends to remain motionless for hours unless moved. Answer 2 is not the first priority. “Touch” is not used in this stage, as in Answer 3. Answer 1 is incorrect because the client is mute and also because intellectual discussion of predisposing factors ignores the feelings of the client.
Reference: DAVIS’S NLCEX RN Success Second Edition, page 463
50. A client who is overweight is referred by the physician to the nurse for diet counselling. What action would the nurse take?
a. Support the client’s interest in other activities.
b. Ask the client to describe his or her eating patterns
c. Develop a weight control plan, together within the client, that will allow gradual weight loss.
d. Put the client on a diet with very limited number of calories so he or she will have an immediate weight loss.
RATIONALE:(3) The nurse should formulate a weight control plan, in cooperation with the client, that allows for a gradual weight loss, not immediate loss as in Answer 4. Answer 1 and 2 are incorporated into Answer 1.
Reference: DAVIS’S NLCEX RN Success Second Edition, page 463


Psychiatric Nursing
(Green highlight indicate that the questions are originally made by me)
1. Francis, a junior nursing student, was with his friends when he received word that his father lost his job and cannot support him in college. At school his professor asks what will be his plans for the future, he answered “I will think about it.”
a. Denial
b. Suppression
c. Projection
d. Insulation
Answer: B – Suppression means the conscious, deliberate forgetting of the unacceptable painful thoughts ideas and feelings --- Saunders Comprehensive Review for the NCLEX-RN Examination, 4th Edition; Linda Ann Silvestri; Pg.1136
2. A 35 year old father went to a pediatric clinic with his 1 year old child for a check-up. The nurse observes that the father of the client was anxious. To help him be aware of this anxiety, the nurse may ask the following questions except:
a. Is something bothering you?
b. Are you comfortable?
c. Are you upset about something?
d. Why are you restless?
Answer: D – Asking the client “Why?” is a non-therapeutic --- Saunders Comprehensive Review for the NCLEX-RN Examination 4th Edition; Linda Ann Silvestri, MSN, RN; Pg.1135
3. A client in a psychiatric unit is imminently aggressive and dangerous not only to himself but to the other patient. The physician order for him to be secluded from the other patient. The nurse should know that the goal of Seclusion is to:
a. Decrease stimulation
b. Provides privacy to the client
c. Give the client to regain self-control
d. Increased protection and prevent property destruction
Answer: C – Seclusion provides decreased stimulation, increased protection, prevention of property damage, and privacy. The goal is to give the client the opportunity to regain physical and emotional self-control --- Psychiatric Mental Health Nursing 2nd Edition; Sheila Videbeck, PhD, RN; Pg.181-182
4. Following the above situation, the nurse should know that the use of seclusion and as well as restraints: except
a. Requires a physician’s order 24 hours ---12 hours
b. Assessment by the nurse every 2-4 hours
c. Nurse checks the client at least 10-15 minutes in person
d. Close supervision and continuous monitoring of the client via video camera
Answer: A –Psychiatric Mental Health Nursing 2nd Edition; Sheila Videbeck, PhD, RN; Pg.181
5. In developing a therapeutic nurse-client relationship, what characteristic should be needed: SATA
a. Honesty
b. Trust
c. Respect
d. Genuineness
Answer: CD – Genuineness, respect, and empathic understanding are characteristics important to the development of a therapeutic nurse-client relationship --- Saunders Comprehensive Review for the NCLEX-RN Examination 4th Edition; Linda Ann Silvestri, MSN, RN; Pg.1133
6. A 28 year old female client was rescued from her abusive husband. While working with her, the nurse should avoid:
a. Minimize the impact of violence
b. Say “I am sorry you have been hurt”
c. Say “the abuse is not your fault”
d. Recommend a support group or individual counseling
Answer: A – Psychiatric Mental Health Nursing 2nd Edition; Sheila Videbeck, PhD, RN; Pg.214
7. A raped client often verbalizes that she was afraid she would be killed during the rape and wish that she had been. The client needs further counseling when she verbalizes the following:
a. “I wish the gave the death penalty to all rapists”
b. “I get so angry at times that I have to have a couple of drinks before I sleep”
c. “Suicide would be an easy escape from all this pain, but I couldn’t do it to myself”
d. “I didn’t fight him, but I guess I did the right thing because I’m alive”
Answer: B – Use of alcohol reflects unhealthy coping mechanisms. A client’s report of needing to calm down needs to be addressed --- Lippincott’s Review for NCLEX-RN 8th Edition; Diane Billings, EdD, RN, FAAN; Pg.639 [#54]
8. A 24 year old female client went to the emergency department complaining of constipation and GI disturbances. Upon assessment it was found out that she has an eating disorder. She was diagnosed of having Compulsive overeating disorder. A nurse should know that Compulsive overeating disorder is:
a. Binge-like overeating without purging
b. Binge-like overeating with purging
c. Life threatening
d. Uncontrolled eating habits
Answer: A – Saunders Comprehensive Review for the NCLEX-RN Examination 4th Edition; Linda Ann Silvestri, MSN, RN; Pg.1172
9. Following the situation above situation, which of the following nursing diagnosis is the most appropriate:
a. Nutrition: Imbalance, more than body requirements
b. Nutrition: Imbalance, less than body requirements
c. Anxiety, death
d. Coping, defensive
Answer: A – If a client is overeating and not purging (like compulsive overeating), then Imbalanced Nutrition: more than body requirements would be an appropriate diagnosis --- Saunders Comprehensive Review for the NCLEX-RN Examination 4th Edition; Linda Ann Silvestri, MSN, RN; Pg.1172 and Lippincott’s Review for NCLEX-RN 8th Edition; Diane Billings, EdD, RN, FAAN; Pg.640 [#70]
10. Kenneth an 18 year old client has been diagnosed of having an eating disorder. Which of the following is not a characteristic of a client having Anorexia Nervosa?
a. An intense fear of being fat
b. Body weight less than 75% of normal ---85%
c. A severely distorted body image
d. Refusal to eat or binge eating and purging
Answer: B – Psychiatric Mental Health Nursing 2nd Edition; Sheila Videbeck, PhD, RN; Pg.453
11. While giving a discharge teaching to the client about lithium therapy, the nurse determines that the teaching has been effective when the client state that he will call the health provider when he experience the following: SATA
a. Muscle weakness
b. Vertigo
c. Fine hand tremor
d. Anorexia
e. Nausea
f. Vomiting
Answer: ABF – Serious side effects that may indicate lithium toxicity include vomiting, extreme hand tremor, sedation, muscle weakness, and vertigo. When lithium is initiated, mild or transient side effects can occur such as fine hand tremors, increased thirst and urination, nausea, anorexia, and diarrhea or constipation --- Lippincott’s Review for NCLEX-RN 8th Edition; Diane Billings, EdD, RN, FAAN; Pg.585 [#60]
12. A nurse is teaching a client about the use of benzodiazepines. Which statement made by the client would indicate further teaching?
a. “Valium will help my tight muscles fell better.”
b. “Valium can make me feel drowsy, so I shouldn’t drive for a while.”
c. “I can’t take drink alcohol while taking Valium.”
d. “I can stop taking the drug anytime I want.”
Answer: D – Benzodiazepines cannot be stopped abruptly. --- Lippincott’s Review for NCLEX-RN 8th Edition; Diane Billings, EdD, RN, FAAN; Pg.620 [#111]
13. A client was diagnosed of having social phobia. What is the best drug of choice for this condition:
a. Alprazolam (Xanax)
b. Chlordiaxepoxide (Librium)
c. Clonazepam (Klonopin)
d. Diazepam (Valium)
Answer: A – Alprazolam (Xanax) is given to a Social Phobia adult PO 2-8mg/day --- Psychiatric Mental Health Nursing 2nd Edition; Sheila Videbeck, PhD, RN; Pg.277 and Mosby’s Drug Guide for Nurses 7th Edition; Linda Skidmore-Roth, RN, MSN, NP; Pg. 62-63
14. While teaching the family of the client with panic disorder, the nurse should include the following in her teaching: SATA
a. Discuss positive coping strategies
b. Emphasize the importance of maintaining prescribed medication regimen
c. Describe time management techniques such as creating a “not to do” list
d. Stress the importance of maintaining contact with community organizations
Answer: ABD – Psychiatric Mental Health Nursing 2nd Edition; Sheila Videbeck, PhD, RN; Pg.283
15. A 34 year old client is suspected to have a conversion disorder. Upon assessment, what will prove that the client has this kind of disorder?
a. Physical complaints of pain and denial of emotional problems ---Somatization disorder
b. Unconcerned with symptoms --- also known as La belle indifference
c. Preoccupation with physical functioning ---Hypochondriasis
d. Anxiety
Answer: C – Saunders Comprehensive Review for the NCLEX-RN Examination 4th Edition; Linda Ann Silvestri, MSN, RN; Pg.1154
16. A 45 year old client has a maintenance medication of Lithium Bicarbonate for his acute manic episodes. The nurse knows that the maintenance margin should be:
a. 0.5-1.2 mEq/L
b. 0.5-1.5 mEq/L
c. 0.6-1.2 mEq/L
d. 0.6-1.5 mEq/L
Answer: B – It takes 10-21 days to achieve a lithium level within the therapeutic range. During an acute manic episode, the normal therapeutic range is 0.5-1.5 mEq/L and the maintenance margin is 0.6-1.2 mEq/L --- Lippincott’s Review for NCLEX-RN 8th Edition; Diane Billings, EdD, RN, FAAN; Pg.565 [#57]
17. The nurse will be giving valproate (Depakote) to a client with bipolar disorder, manic phase. Before giving the medication, the nurse check the client’s valproate level. What is the therapeutic level of valproate (Depakote)?
a. 10-15 ug/mL
b. 25-50 ug/mL
c. 45-75 ug/mL
d. 50-100 ug/mL
Answer: D – Lippincott’s Review for NCLEX-RN 8th Edition; Diane Billings, EdD, RN, FAAN; Pg.566 [#66]
18. A 55 year old man has been diagnosed of having schizophrenia, paranoid type. Which of the following is not a negative symptom of schizophrenia?
a. Bizarre behavior
b. Inability to experience pleasure
c. Poverty of thought
d. Loss of motivation
Answer: A – The Positive Symptoms of Schizophrenia are hallucinations, delusions, disorganized speech, and bizarre behavior, while the negative symptoms are blunted affect, poverty of thought (alogia), loss of motivation (avolition), and inability to experience pleasure or joy (anhedonia) --- Saunders Comprehensive Review for the NCLEX-RN Examination 4th Edition; Linda Ann Silvestri, MSN, RN; Pg.1157
19. A 48 year old client with a long history of schizophrenia, paranoid type is readmitted for not taking his last two injections of haloperidol on purpose. He reports, “I’m not sleeping much and my friend say I stink from not taking a shower for a week. God is telling me not to trust others because they will just hurt me. My parents don’t want me anymore and wish that I was dead.” Upon admissions which of the following nurse by the nurse contains assumptions and false accusations? SATA
a. Client has missed two doses of Haldol Decanoate. He’s not sleeping and showering. Has a strained relationship with his parents and delusions that they want him dead. Voluntary admission to restart Haldol Decanoate.
b. Client ha been noncompliant with his medications causing decreased sleep and activities of daily living, increased auditory hallucinations, and paranoid about his parents harming him.
c. Client admitted for noncompliance with Haldol Decanoate injections, sleep disturbance, poor hygiene, auditory hallucinations, and suspiciousness of his parents. Needs to be monitored for suicidal and homicidal ideation.
d. Client has missed two injections of Haldol Decanoate and was admitted voluntarily. He reports he has decreased sleep and showering and that he hears God’s voice telling him to protect himself from others. He stated “My parents don’t want me and wish that I was dead.”
Answer: ABC – Documentation provided in letter D is the most factual and without conclusions or assumptions --- Lippincott’s Review for NCLEX-RN 8th Edition; Diane Billings, EdD, RN, FAAN; Pg.587 [#22]
20. A 4 year old client was brought in the emergency department by his mother, reports that her son fell while climbing the stairs. Upon assessment, the nurse found unexplained bruises all over his body and bald spots on the scalp. Which of the following would the nurse prioritize to do?
a. Assess injuries; support the child during a thorough physical assessment
b. Document accurately and completely all information related to the suspected abuse
c. Move slowly and avoid any loud noises when near the child
d. Report cases of suspected abuse to appropriate authorities
e. Reassure the client that he or she is not “bad” and is not responsible for the abuser’s behavior
f. Communicate with the child at the child’s eye level
21. Arrange the above according to prioritization:
Answer: ADCFEB – Saunders Comprehensive Review for the NCLEX-RN Examination 4th Edition; Linda Ann Silvestri, MSN, RN; Pg.1192
22. The nurse is assessing the client’s coping methods. A client who is being abused would demonstrate the following: SATA
a. Self-blame
b. Suicidal thoughts
c. Assertiveness
d. Alcohol abuse
Answer: ABD – Self-blame, substance abuse, and suicidal thoughts and attempts are possible dysfunctional coping methods used by abuse victims --- Lippincott’s Review for NCLEX-RN 8th Edition; Diane Billings, EdD, RN, FAAN; Pg.566 [#66]
23. The mother of a 4 year old client with ADHD asks the nurse when will the drug, methylphenidate (Ritalin), take its full effect:
a. After 24 hours
b. After 32 hours
c. After 48 hours
d. After 72 hours
Answer: C – One of the nursing considerations is to alert the client that the full drug effect takes 2 days (48 hours) --- Psychiatric Mental Health Nursing 2nd Edition; Sheila Videbeck, PhD, RN; Pg.488
24. A 3 year old child is brought to the child’s pediatrician by her parents who are concerned by her behavior. They state that she always do incessant activities with difficulty completing tasks. Which of the following would the nurse suspect?
a. ADHD incessant
b. Schizophrenia
c. Autism
d. Tourette syndrome
Answer: C – ADHD is most often portrayed as incessant activity with difficulty completing tasks --- Lippincott’s Review for NCLEX-RN 8th Edition; Diane Billings, EdD, RN, FAAN; Pg.641 [#73]
25. A client is admitted to a psychiatric institution for numerous incidents of threatening, angry outbursts, and two episodes of hitting a coworker at the call center where he works. The client is very anxious and tells the nurse who admits him, “I didn’t mean to hit him. He mad me so angry that I can’t control myself to hit him. I hope I don’t hit anymore.” What will be the initial action to make to ensure safe environment?
a. Obtaining an order for a medication to be administered to decrease his anxiety and threatening behavior
b. Letting other clients know that he has a history of hitting others so that they will not provoke him
c. Putting him in a private room and limiting his time out of the room to when staff ca be with him
d. Telling him that hitting others is unacceptable behavior and asking him to tell a staff member when he begins feeling angry
Answer: D – The nurse must clearly address behavioral expectations, such as telling the client that hitting is unacceptable and also provide alternatives for the client, such as letting staff members know when he begins to feel angry --- Lippincott’s Review for NCLEX-RN 8th Edition; Diane Billings, EdD, RN, FAAN; Pg.636 [#21]
26. Eddie, a 16 year old client, broke a chair during an angry outburst. Which response will be appropriate:
a. “What’s the matter with you? Don’t you know any better?”
b. “Why are you doing this?”
c. “Eddie, breaking chairs is unacceptable behavior. You need to let the staff know you’re upset so you can talk about it instead of acting out.”
d. “Don’t ever do that again or else you will be punish”
Answer: C – Psychiatric Mental Health Nursing 2nd Edition; Sheila Videbeck, PhD, RN; Pg.498
27. A client went to the psychiatric unit complaining of having emotional instability, mood swings and feeling depress. She is diagnosed of having a Personality disorder with symptoms of Mood dysregulation. Emotional instability and mood swings respond favorably to which of the following drug of choice:
a. Cholinergic agonists (donepezil)---for organic-like depression
b. Carbamazepine (Tegretol) ---can also be use for Ictal depression (abnormal)
c. Imipramine (Tofranil) ---for organic-like depression
d. Diphenylhydantoin (Dilantin) ---for Ictal depression (abnormal)
Answer: B – Emotional Instability and mood swings respond favorably to Lithium, carbamazepine (Tegretol), valproate (Depakote), or low-dose neuroleptics such as haloperidol (Haldol) --- Psychiatric Mental Health Nursing 2nd Edition; Sheila Videbeck, PhD, RN; Pg.378-379
28. Which of the following nursing intervention is appropriate to a client with personality disorder, schizotypal type?
a. Improve client’s functioning in the community; assist client to find case manager ---Schizoid type
b. Teach social skills; provide factual feedback about behavior ---Histrionic type
c. Support and reassurance; cognitive restructuring techniques; promote self-esteem ---Avoidant type
d. Develop self-care skills; improve community functioning; social skills training
Answer: C – Psychiatric Mental Health Nursing 2nd Edition; Sheila Videbeck, PhD, RN; Pg.380
29. A client who had been living with her older sister after her boyfriend of 1 year told her to leave is admitted to the subacute unit complaining of feeling empty and lonely, being unable to sleep and hardly eating for the past two weeks. Her arms are full of wounds and scars from frequent self-mutilation. Which kind of personality disorder is this?
a. Compulsive personality disorder
b. Antisocial personality disorder
c. Borderline personality disorder
d. Avoidant personality disorder
Answer: D – The client’s signs and symptoms are indicative of borderline personality disorder, by impulsivity, self-mutilating behavior, unstable and intense personal relationships, identity disturbance, chronic feeling of emptiness, frantic avoidant of abandonment, and problems with anger --- Lippincott’s Review for NCLEX-RN 8th Edition; Diane Billings, EdD, RN, FAAN; Pg.609 [#10]
30. A client with symptoms of alcohol withdrawal is given chlordiazepoxide (Librium) 400mg/day, repeat in 2-4 hours if necessary. Before giving the medication, the nurse should:
a. Withhold giving the drug because the dosage is too low
b. Withhold giving the drug because the dosage is too high
c. Give the medication on the right time
d. Check the vital signs before giving the medication
Answer: B – Psychiatric Mental Health Nursing 2nd Edition; Sheila Videbeck, PhD, RN; Pg.420
31. In teaching the client about taking chlordiazepoxide (Librium),an anti-anxiety drug, the nurse should teach the client to avoid the following drug/herb to avoid increasing the effect of the drug: SATA
a. Cowslip
b. Kava
c. Queen Anne’s Lace
d. Valerian
Answer: ABCD – Mosby’s Drug Guide for Nurses 7th Edition; Linda Skidmore-Roth, RN, MSN, NP; Pg. 218-219
32. Which of the following nursing implication is incorrect regarding the use of fluoxetine (Prozac)?
a. Administer either in AM or PM
b. Monitor for hyponatremia
c. Monitor for hypokalemia
d. Encourage adequate fluids
Answer: C – Administer in AM if nervous or PM if drowsy, also report sexual difficulties to the physician --- Psychiatric Mental Health Nursing 2nd Edition; Sheila Videbeck, PhD, RN; Pg.338
33. The physician of a 2 year old client with ADHD prescribed pemoline (Cylert) 37.5 mg OD. The mother of the client asks the nurse how long it will take to see changes to his son. The nurse knows that pemoline (Cylert) will take full effect after:
a. 2 days
b. 5 days
c. 7 days
d. 14 days
Answer: D – pemoline (Cylert): dosage (mg/day) 37.5-112.5 in the morning: Alert client that the drug may take 2 weeks to achieve its full effect --- Psychiatric Mental Health Nursing 2nd Edition; Sheila Videbeck, PhD, RN; Pg.488
34. Which of the following teaching to the parents of a child with ADHD is incorrect:
a. Include parents in planning and providing care
b. Focus on the child’s strengths and not the problems/weakness
c. Teach accurate administration of medication and its side effects
d. Refer parents to support groups
Answer: B – Focus on the child’s strengths as well as problems --- Psychiatric Mental Health Nursing 2nd Edition; Sheila Videbeck, PhD, RN; Pg.491
35. A 24 year old client with severe anxiety was brought by her parents to a psychiatric hospital. As a nurse, which of the following intervention should be done to decrease the level of her anxiety? SATA
a. Use a calm manner
b. Always remain with the client
c. Provide fine motor activity ---gross motor activity
d. Provide clear, simple statements
Answer: C – Saunders Comprehensive Review for the NCLEX-RN Examination 4th Edition; Linda Ann Silvestri, MSN, RN; Pg.1152
36. The mother of a 16 year old female brought her daughter to a psychiatric unit stating, “My daughter won’t stop cleaning her room and taking so long to bathe. She’s afraid of going out, saying that she will just get sick when she goes out because there are a lot of dirt and germs outside our house.” The nurse knows that these are symptoms of:
a. Astraphobia ---fear of electrical storms
b. Nyctophobia ---fear of darkness
c. Pyrophobia ---fear of fires
d. Mysophobia ---fear of dirt or germs
Answer: D – Saunders Comprehensive Review for the NCLEX-RN Examination 4th Edition; Linda Ann Silvestri, MSN, RN; Pg.1153
37. Which of the following interventions for an Obsessive-Compulsive client is correct:
a. Be sympathetic toward the client ---empathetic
b. Do not interrupt the compulsive behavior
c. Establish a verbal contract that will assist the client to decrease the frequency of compulsive behaviors gradually ---written
d. Recognize and reinforce positive ritualistic behaviors ---nonritualistic
Answer: C – Saunders Comprehensive Review for the NCLEX-RN Examination 4th Edition; Linda Ann Silvestri, MSN, RN; Pg.1153
38. In assessing a 45 year old male client, which of the following is not a sign/symptom of alcohol abuse:
a. Belligerence
b. Binge drinking
c. Experiencing blackouts
d. Blood alcohol levels of 0.05% or higher ---0.1%
Answer: D – Saunders Comprehensive Review for the NCLEX-RN Examination 4th Edition; Linda Ann Silvestri, MSN, RN; Pg.1174
39. A client developed withdrawal delirium 72 hours after cessation of drinking alcohol. Withdrawal delirium is a medical emergency thus a client needs careful monitoring. The mother of the client asks the nurse how long will the delirium lasts. The nurse knows that the delirium will lasts for:
a. 1-2days
b. 2-3days
c. 3-5days
d. 5-7days
Answer: B – The state of delirium usually peaks 48-72 hours after cessation or reduction of intake (although can occur later) and lasts 2-3 days --- Saunders Comprehensive Review for the NCLEX-RN Examination 4th Edition; Linda Ann Silvestri, MSN, RN; Pg.1175
40. The client states that he’s taking citalopram (Celexa) 40 mg everyday as prescribed by his physician. He also states that he’s also been taking St. John’s wort 750 mg daily for 2 weeks now. Which of the following signs/symptoms suggest that the client is developing serotonin syndrome? SATA
a. Restlessness
b. Ataxia
c. Diaphoresis
d. Headache
e. Confusion
f. Constipation
Answer: ABCDE – Serotonin syndrome can occur if a SSRI is combined with MAOI, a tryptophan-serotonin precursor, or St. John’s wort. Signs and symptoms of serotonin syndrome includes mental status changes such as confusion, restlessness or agitation, myoclonus, hyperreflexia, diaphoresis, shivering, tremor, diarrhea, nausea, abdominal cramps, ataxia and headache. Constipation is not associated with serotonin syndrome --- Lippincott’s Review for NCLEX-RN 8th Edition; Diane Billings, EdD, RN, FAAN; Pg.609 [#21]
41. A client went to the ED with complains of blurring of vision after taking haloperidol (Haldol), buspirone (BuSpar), quetiapine (Seroquel), and benztropine (Cogentin) for 4 days. Which of the following medications has this side effect?
a. Haloperidol (Haldol)
b. Buspirone (BuSpar)
c. Quetiapine (Seroquel)
d. Benztropine (Cogentin)
Answer: D – benztropine (Cogentin) frequently causes the side effect of blurred vision. Quetiapine (Seroquel), an atypical antipsychotic, and buspirone (BuSpar), an anti-anxiety agent, are not likely to produce blurred vision. Although haloperidol (Haldol), a high-potency anti-psychotic, may cause blurred vision, this side effect is more common with benztropine --- Lippincott’s Review for NCLEX-RN 8th Edition; Diane Billings, EdD, RN, FAAN; Pg.609 [#12]
42. In planning for the care of a client with delirium, which of the following should be prioritize based on the nurse’s understanding about the disturbances in consciousness associated with this disorder?
a. Eliminating the client’s napping in the daytime as much as possible
b. Avoiding arguing with a suspicious client about perception of reality
c. Identifying self and make sure that the nurse has the client’s attention
d. Engaging the client in reminiscing with relatives or visitors
Answer: C – Identifying one’s self and making sure that the nurse has the client’s attention addresses the difficulties with focusing and maintaining attention. Eliminating daytime napping is unrealistic until the cause of delirium is determined and the client’s ability to focus and maintain attention improves. Engaging the client in reminiscing and avoiding arguing are also unrealistic at this time --- Lippincott’s Review for NCLEX-RN 8th Edition; Diane Billings, EdD, RN, FAAN; Pg.609 [#89]
43. A client is brought in the ED by his sister who states, “He’s been using so many heroins until he ran out of money about 48 hours ago.” Which of the following are signs and symptoms of opioid withdrawal? SATA
a. Formication
b. Piloerection
c. Diaphoresis
d. Rhinorrhea
e. Synesthesia
Answer: BCD – Symptoms of opioid withdrawal include yawning, rhinorrhea, sweating, chills, piloerection (goose bumps), tremor, restlessness, irritability, leg spasms, bone pain, diarrhea, and vomiting. Symptoms of withdrawal occur within 36-72 hours of usage and subside within a week. Withdrawal from heroine is seldom fatal and usually does not necessitate medical intervention. Synesthesia (a blending of senses) is associated with LSD use, and formication (feeling of bugs crawling beneath the skin) is associated with cocaine use --- Lippincott’s Review for NCLEX-RN 8th Edition; Diane Billings, EdD, RN, FAAN; Pg.609 [#65]
44. What information would the school nurse include when developing a teaching plan for a group of middle school children about the drug, Ecstasy? SATA
a. It reduces self-consciousness
b. Ecstasy is used at “raves”
c. Candy-like pacifiers are used for teeth grinding
d. Using Ecstasy is similar to using speed
e. It can cause death
Answer: ALL – Ecstasy is chemically related to methamphetamine (speed) and is use at “raves” (all-night dance parties) to enhance dancing, closeness to others, affection, and the ability to communicate. Euphoria, heightened sexuality, disinhibition, and decrease self-consciousness can occur. Adverse effects include tachycardia, elevated blood pressure, anorexia, dry mouth, and teeth grinding. Pacifiers, including candy-shaped pacifiers and lollipops, are used to ease the discomfort associated with teeth grinding and jaw clenching. Hyperthermia, dehydration, renal failure, and death can occur --- Lippincott’s Review for NCLEX-RN 8th Edition; Diane Billings, EdD, RN, FAAN; Pg.609 [#65]
45. In dealing with a client with De-Escalation techniques associated with Bipolar Disorder, which nursing implication is incorrect?
a. Maintain large personal space and use a nonaggressive posture
b. Avoid verbal struggles
c. Encourage client to talk out instead of acting out feelings of frustration ---Aggressive Behavior
d. Determine what the client considers his or her need
Answer: C – Saunders Comprehensive Review for the NCLEX-RN Examination 4th Edition; Linda Ann Silvestri, MSN, RN; Pg.1156
46. Three months after the death of his wife in an automobile accident, a client is admitted to the psychiatric hospital after attempting to overdose on his anti-depressant medication. He states “I can’t live without his wife. It’s no use and I just want to die.” Which nursing diagnosis is appropriate?
a. Risk for Self-Directed Violence related to wife’s death as evidenced by the client’s wish to die
b. Dysfunctional Grieving related to wife’s death as evidenced by a suicide attempt
c. Hopelessness related to wife’s death as evidenced by the client’s statement of inability to live without the wife
d. Powerlessness related to wife’s death as evidenced by statement of “It’s no use”
Answer: B – Risk for Self-Directed Violence is the priority nursing diagnosis for a client who attempted or verbalizes the intent to harm himself. Although the client is depressed and feeling hopeless, and grieving, these are not the priority concern at this time --- Lippincott’s Review for NCLEX-RN 8th Edition; Diane Billings, EdD, RN, FAAN; Pg.635 [#8]
47. The nurse knows which of the following in assessing children and adolescent for suicidal risk?
a. Adolescents typically don’t choose suicide unless they live in a certain geographical regions of the United States including the western states
b. The risk of suicide increases during adolescence, with those who have recently suffered a loss, abuse, or family discord being most at risk
c. Children rarely commit suicide unless one of their parents has already committed suicide, especially in the past year
d. Children do have suicidal risk that coincide with some significant events such as a recent gun purchase in the family
Answer: B – Adolescents are amore likely than children to attempt or commit suicide. Loss, abuse, and family discord remain significant risk factors. There is no evidence to support that children rarely commit suicide. Additionally, evidence fails to support the belief that children who have lost a parent to suicide will attempt it themselves. Significant events, such as recent firearm purchase, have not been linked to suicide attempts in children. No geographical region in the united States is free from adolescent suicide --- Lippincott’s Review for NCLEX-RN 8th Edition; Diane Billings, EdD, RN, FAAN; Pg.641 [#80]
48. In providing the best care for the family and client with Asperger’s disorder, the nurse knows that this disorder is different from autism in which of the following areas?
a. In Asperger’s disorder, behavior often is similar to that of other children with autism but without the problems with school
b. There are significant problems with language development, as with autism, but there are no delays or difficulties with motor development
c. Asperger’s disorders, often diagnosed earlier than autism, is associated with fewer major problems in interpersonal interaction
d. Asperger’s disorder is recognized later than autism, and interpersonal interaction problems typically become more apparent when the child begins school
Answer: D – Asperger’s disorder is recognized later than autism, and the interpersonal problems worsen with school attendance. These children usually have restricted ant repetitive patterns of behavior. School problems exist as a result of interaction difficulties and behavior differences. Motor development may be delayed, but language often progresses normally --- Lippincott’s Review for NCLEX-RN 8th Edition; Diane Billings, EdD, RN, FAAN; Pg.641 [#82]
49. The nurse is checking the laboratory results of the client. The Imipramine level of the client is within normal range. This indicates that the client’s serum concentration is within which of the following ranges?
a. 351-450 mg/mL
b. 251-350 mg/mL
c. 151-250 mg/mL
d. 51-150 mg/mL
Answer: C – the therapeutic serum concentration level for Imipramine is 151-250 mg/mL. at the upper limit of the therapeutic range, serious cardiac and central nervous system effects may begin to develop --- Lippincott’s Review for NCLEX-RN 8th Edition; Diane Billings, EdD, RN, FAAN; Pg.609 [#31]
50. Which of the following would the nurse identify as the characteristic found in AD that distinguishes it from dementia when describing AD to a group of nursing students?
a. An infectious particle called a prion
b. Hyperkinesis causing choreiform movements
c. Hypoxic destruction of brain cells
d. Neurofibrillary tangles and plaques
Answer: D – Neurofibrillary tangles and plaques are found in postmortem examinations of AD clients, distinguishing it from other dementias. Hypoxic destruction of brain cells is associated with Huntington’s disease. Evidence of an infectious particle called prion is associated with Creutzfeldt-Jakob disease --- Lippincott’s Review for NCLEX-RN 8th Edition; Diane Billings, EdD, RN, FAAN; Pg.593 [#97]